You are on page 1of 50

ĐẠI HỌC THÁI NGUYÊN

TRƯỜNG ĐẠI HỌC KHOA HỌC

ĐỖ PHƯƠNG THẢO

VỀ BÀI TOÁN CỰC TRỊ


TRONG HÌNH HỌC TỔ HỢP

LUẬN VĂN THẠC SĨ TOÁN HỌC

THÁI NGUYÊN-2019
ĐẠI HỌC THÁI NGUYÊN
TRƯỜNG ĐẠI HỌC KHOA HỌC

ĐỖ PHƯƠNG THẢO

VỀ BÀI TOÁN CỰC TRỊ


TRONG HÌNH HỌC TỔ HỢP

CHUYÊN NGÀNH: PHƯƠNG PHÁP TOÁN SƠ CẤP


MÃ SỐ: 8 46 01 13

LUẬN VĂN THẠC SĨ TOÁN HỌC

NGƯỜI HƯỚNG DẪN KHOA HỌC

PGS. TS. TRỊNH THANH HẢI

THÁI NGUYÊN-2019
Mục lục
Trang

Mở đầu 1

Chương 1 Kiến thức chuẩn bị 3


1.1 Tổng quan về bài toán hình học tổ hợp . . . . . . . . . . . . . . 3
1.2 Một số nguyên lý, phương pháp giải toán thường gặp trong lời
giải các bài toán hình học tổ hợp . . . . . . . . . . . . . . . . . . 4
1.2.1 Một số nguyên lý . . . . . . . . . . . . . . . . . . . . . . . 4
1.2.2 Phương pháp đếm hai lần (Double Counting) . . . . . . . 6
1.3 Một số ví dụ về bài toán hình học tổ hợp . . . . . . . . . . . . . 8
1.3.1 Các bài toán đếm trong hình học tổ hợp . . . . . . . . . 8
1.3.2 Các bài toán chứng minh trong hình học tổ hợp . . . . . 8

Chương 2 Một số bài toán về cực trị trong hình học tổ hợp 22
2.1 Bài toán về tìm giá trị lớn nhất . . . . . . . . . . . . . . . . . . . 22
2.2 Bài toán về tìm giá trị nhỏ nhất . . . . . . . . . . . . . . . . . . . 36
2.3 Bài toán liên quan đến cực trị hình học tổ hợp . . . . . . . . . . 43

Kết luận 46

ii
Mở đầu

Từ thời xa xưa vấn đề toán học được ra đời từ rất sớm từ các hoạt động
thực tiễn của con người, trong đó có tư duy về hình học tổ hợp, ví dụ: Ở những
nước châu Á, trong số đó có Ấn Độ, các nhà toán học Jaina đã nghiên cứu ra
dãy số, các dãy cấp số, hoán vị và tổ hợp; Thời Trung Quốc cổ đại, người ta
đã biết đến biểu đồ tổ hợp phức còn gọi là “hình vuông thần kì”; Thời kì cổ
đại ở Hy Lạp đã có những nhà triết học thông thái đặc biệt là nhà triết học
Kxenorat đã biết từ những chữ cái cho trước lập thành bảng chữ số. . . Nhưng
phải đến khoảng thế kỉ XVII – XVIII với những công trình nghiên của như
Pascal, Fermat, Euler. . . thì toán học tổ hợp mới thực sự hình thành như một
nhánh của toán học. Toán tổ hợp có tính hấp dẫn, lý thú của toán học nói
chung và toán sơ cấp nói riêng. Nội dung của toán tổ hợp phong phú và được
ứng dụng nhiều trong thực tế đời sống. . .
Hình học tổ hợp là một nhánh không thể thiếu toán tổ hợp, là những bài
toán hay, thú vị và thường xuyên xuất hiện trong các cuộc thi học sinh giỏi
Quốc gia, Olympic toán quốc tế, thi Olympic sinh viên giữa các trường đại học,
cao đẳng trong cả nước. Ở Trường Đại học Khoa học- Đại học Thái Nguyên
đã có học viên Lê Thị Bình đã làm luận văn Thạc sĩ với đề tài “Các bài toán
về hình học tổ hợp” nhưng chưa luận văn đề cập một cách hệ thống đến dạng
toán “Cực trị trong hình học tổ hợp”. Chính vì với mong muốn tìn hiểu sâu về
các toán cực trị trong hình học tổ hợp, em đã chọn đề tài “Các bài toán cực
trị hình học tổ hợp” làm đề tài luận văn thạc sĩ của mình.
Mục đích nghiên cứu của luận văn được xác định là: Sưu tầm, nghiên cứu
và trình bày một cách có chọn lọc về bài toán cực trị trong hình học tổ hợp
để hình thành một tài liệu giảng dạy chuyên đề bồi dưỡng học sinh khá, giỏi.
Nội dung chính của luận văn được trình bày thành hai chương:
Chương 1: Trong chương này, luận văn trình bày một số nguyên lý và phương
pháp thường gặp trong các lời giải của bài toán hình học tổ hợp, kèm theo các
ví dụ, các bài tập minh họa.
Chương 2: Nội dung chương 2 được dành riêng để trình bày lời giải của một
số bài toán cực trị hình học tổ hợp dành cho học sinh khá, giỏi và được sắp

1
xếp theo hai dạng chính là: Bài toán liên quan đến tìm giá trị lớn nhất, tìm
giá trị nhỏ nhất trong hình học tổ hợp.
Luận văn này được hoàn thành tại trường Đại học Khoa học, Đại học Thái
Nguyên dưới sự hướng dẫn tận tình của PGS. TS Trịnh Thanh Hải. Em chân
thành cảm ơn thầy Trịnh Thanh Hải đã tận tình hướng dẫn em triển khai đề
tài của luận văn này. Em xin chân thành cảm ơn gia đình, bạn bè cùng các
anh chị đã tạo điều kiện để em hoàn thành đề tài này.
Tuy nhiên điều kiện về năng lực bản thân còn hạn chế, luận văn chắc chắn
không tránh khỏi những thiếu sót. Kính mong nhận được sự đóng góp ý kiến
của các thầy cô giáo, bạn bè và đồng nghiệp để bài luận văn của em được hoàn
thiện hơn.
Em xin trân trọng cảm ơn!

Thái Nguyên, tháng 05 năm 2019

Học viên

Đỗ Phương Thảo

2
Chương 1

Kiến thức chuẩn bị

1.1 Tổng quan về bài toán hình học tổ hợp


Trước tiên, luận văn xin nhắc lại một vài dạng toán tổ hợp được trình bày
trong luận văn:
(i) Bài toán cực trị tổ hợp:
Dạng 1: Tìm số nguyên dương k nhỏ nhất (lớn nhất) sao cho mọi tập A
mà | A |= k là hữu hạn đều có tính chất T nào đó.
Ví dụ 1.1.1. Gọi A là tập tất cả các số tự nhiên lẻ không chia hết cho 5 và
nhỏ hơn 30. Tìm số k nhỏ nhất sao cho mỗi tập con của A gồm k phần tử đều
tồn tại hai số chia hết cho nhau?
Ví dụ 1.1.2. Cho tập A gồm 16 số nguyên dương đầu tiên. Hãy tìm số nguyên
dương k nhỏ nhất có tính chất: Trong mỗi tập con có k phần tử của A đều tồn
tại hai số phân biệt a, b sao cho a2 + b2 là số nguyên tố (VMO 2004).
Với bài toán dạng này, chúng ta thường xét một tập A có tính chất đặc
biệt nào đó sao cho | A |= m và A không thỏa mãn tính chất T , từ đó suy ra
được kmin > m + 1. Tiếp theo ta chứng minh mọi tập A mà | A |= m + 1 đều
có tính chất T , từ đó ta tìm được kmin = m + 1. Để chứng minh mọi tập A mà
| A |= m + 1 đều có tính chất T thì ta có thể sử dụng nguyên lí Dirichlet hoặc
dựa vào tính chất tập A.
Dạng 2: Tìm số phần tử lớn nhất (nhỏ nhất) của tập A gồm các phần tử
có tính chất T .
Ví dụ 1.1.3. Cho A là tập hợp gồm 8 phần tử. Tìm số lớn nhất các tập con
gồm 3 phần tử của A sao cho giao của hai tập bất kì trong các tập con này
không phải là tập gồm hai phần tử.
Ví dụ 1.1.4. Trong một cuộc thi có 11 thí sinh tham gia giải 9 bài toán. Hai
thí sinh bất kì giải chung với nhau không quá một bài. Tìm k lớn nhất để mọi
bài toán có ít nhất k thí sinh giải được.

3
Để giải bài toán này, chúng ta thường thực hiện theo cách sau:
Đặt | A |= k , bằng các lập luận ta chứng minh k < m (k > m). Sau đó ta xây
0 0
dựng một tập A thỏa tính chất T và | A |= m.

(ii) Bài toán cực trị hình học tổ hợp


Các bài toán cực trị tổ hợp (i), mà tập A gồm các đối tượng hình học thì
thường được xếp vào dạng Bài toán cực trị hình học tổ hợp.

Ví dụ 1.1.5. Cho một đa giác đều 2007 đỉnh. Tìm số nguyên dương k nhỏ
nhất thoả mãn tính chất: Trong mỗi cách chọn k đỉnh của đa giác, luôn tồn
tại 4 đỉnh tạo thành một tứ giác lồi mà 3 trong số 4 cạnh của tứ giác là cạnh
của đa giác đã cho (VMO 2007).

Ví dụ 1.1.6. Cho 2006 điểm phân biệt trong không gian, không có bốn điểm
nào thẳng hàng. Số k gọi là số tốt nếu ta có thể điền lên mỗi đoạn thẳng nối
hai điểm trong 2006 điểm đã cho một số tự nhiên không vượt quá k sao cho
với mọi tam giác có ba đỉnh trong 2006 điểm đã cho thì có hai cạnh được điền
hai số bằng nhau và cạnh còn lại thì được điền số lớn hơn. Tìm số tốt có giá
trị nhỏ nhất (TST Việt Nam 2006).

Phương pháp giải các bài toán cực trị hình học tổ hợp này sẽ được luận văn
trình bày chi tiết trong nội dung của Chương 2.

1.2 Một số nguyên lý, phương pháp giải toán thường gặp
trong lời giải các bài toán hình học tổ hợp
1.2.1 Một số nguyên lý

Nguyên lý cộng
Quy tắc cộng: Nếu Ei (i = 1, . . . , k ) với k sự kiện thỏa mãn:

(i) Không có hai sự kiện nào trong số chúng xảy ra đồng thời;

(ii) Ei có thể xảy ra theo ni cách thì một trong k sự kiện có thể xảy ra theo
(n1 + n2 + · · · + nk ) cách.

Nguyên lý nhân
Quy tắc nhân: Nếu Ei (i = 1, . . . , k ) với k sự kiện thỏa mãn. Và E1 có thể
xảy ra theo n1 cách, E2 có thể xảy ra theo n2 cách (không phụ thuộc đến việc
E1 xảy ra như thế nào); E3 có thể xảy ra theo n3 cách (không phụ thuộc đến
việc E2 , E1 xảy ra như thế nào), . . . , Ek có thể xảy ra theo nk cách (không phụ
thuộc đến (k − 1) sự kiện trước xảy ra như thế nào), thì k sự kiện có thể xảy
ra đồng thời theo n1 · n2 · n3 . . . nk cách.

4
Nguyên lý bù trừ
Khi hai công việc có thể được làm đồng thời, ta không thể dùng quy tắc
cộng để tính số cách thực hiện nhiệm vụ gồm cả hai việc. Để tính đúng số cách
thực hiện nhiệm vụ này, ta cộng số cách làm mỗi một trong hai việc rồi trừ
đi số cách làm đồng thời cả hai việc. Ta có thể phát biểu nguyên lý đếm này
bằng ngôn ngữ tập hợp. Cho A1 , A2 là hai tập hữu hạn, khi đó:
[
| A1 A2 |=| A1 | + | A2 | − | A1 ∩ A2 |

Từ đó, với ba tập hữu hạn A1 , A2 , A3 ta có:

| A1 ∪ A2 ∪ A3 |=| A1 | + | A2 | + | A3 | − | A1 ∩ A2 | − | A1 ∩ A3 | − | A3 ∩ A2 | +

| A1 ∩ A2 ∩ A3 | .

Và bằng quy nạp, với k tập hữu hạn A1 , A2 , . . . , Ak ta có:

| A1 ∩ A2 ∩ · · · ∩ Ak |= N1 − N2 + N3 − · · · + (−1)k−1 Nk .

Trong đó Nm (1 ≤ m ≤ k ) là tổng phần tử của tất cả các giao m tập lấy từ k


tập đã cho, nghĩa là:
X
Nm = | Ai1 ∩ Ai2 ∩ · · · ∩ Aim |
1<i1 <i2 <···<im ≤k

Bây giờ, ta đồng nhất tập Am (1 ≤ m ≤ k ) với tính chất Am cho trên tập vũ
trụ hữu hạn U nào đó và đếm xem có bao nhiêu phần tử của U sao cho không
thỏa mãn bất kỳ một tính chất Am nào. Gọi N là số cần đếm, N là số phần
tử của U . Ta có:

N = N − | A1 ∪ A2 ∪ · · · ∪ Ak |= N − N1 + N2 − N3 + · · · + (−1)k Nk .

Trong đó Nm là tổng các phần tử của U thỏa mãn m tính chất lấy từ k tính
chất đã cho.
Nguyên lý cực hạn

NL 1: Trong tập hợp hữu hạn và khác rỗng các số thực luôn có thể chọn được
số bé nhất và số lớn nhất.

NL 2: Trong một tập hợp khác rỗng các số tự nhiên luôn luôn có thể chọn được
số bé nhất.

Nguyên lý trên được gọi là nguyên lý cực hạn. Nhờ nguyên lý này ta có thể
xét các phần tử mà một đại lượng nào đó có giá trị nhỏ nhất hoặc giá trị lớn
nhất, chẳng hạn:

5
• Xét đoạn thẳng có độ dài lớn nhất (nhỏ nhất) trong một số hữu hạn các
đoạn thẳng.
• Xét góc lớn nhất (nhỏ nhất) trong một số hữu hạn góc.

• Xét đa giác có diện tích hoặc chu vi lớn nhất (nhỏ nhất) trong một số hữu
hạn các đa giác.
• Xét khoảng cách lớn nhất (nhỏ nhất) trong một số hữu hạn khoảng cách
giữa hai điểm hoặc khoảng cách từ một điểm đến một đường thẳng.
• Xét các điểm là đầu mút của một đoạn thẳng, xét các điểm ở phía trái
nhất hoặc ở phía phải nhất của một đoạn thẳng (giả thiết là đoạn thẳng
đó nằm ngang).
Nguyên lý Dirichlet
Nguyên lý Dirichlet do nhà toán học người Đức nổi tiếng là Dirichlet đề
xuất từ thế kỷ XX đã được áp dụng để chứng minh sự tồn tại nghiệm trong
nhiều bài toán tổ hợp. Nguyên lý này được phát triển từ một mệnh đề rất đơn
giản gọi là nguyên lý “nguyên lý quả cam” hay là nguyên lý “chuồng chim bồ
câu”: Giả sử có một đàn chim bồ câu bay vào chuồng. Nếu số chim nhiều hơn
số ngăn chuồng thì chắc chắn có ít nhất một ngăn có nhiều hơn một con chim.
Dạng đơn giản nhất của nguyên lý Dirichlet, hay còn gọi là nguyên lý nhốt
thỏ vào lồng, như sau: Nếu nhốt n + 1 thỏ vào n lồng thì tồn tại một lồng có
ít nhất 2 con thỏ. Tổng quát: Nếu n > km (n, k, m) là các số tự nhiên) thì khi
nhốt n con thỏ vào m lồng sẽ tồn tại một lồng chứa ít nhất k + 1 thỏ. Thật
vậy, giả sử lồng nào cũng có không quá k thỏ thì m lồng có không quá mk thỏ,
ít hơn n thỏ, vô lý.

1.2.2 Phương pháp đếm hai lần (Double Counting)

Ý tưởng của phương pháp đếm hai lần: Với bài toán đếm, nếu ta sử dụng
hai phương pháp đếm khác nhau thì kết quả phải trùng nhau.
Ví dụ 1.2.1. [3] Người ta kẻ n đường thẳng, trong đó không có hai đường
thẳng nào song song với ba đường thẳng nào đồng quy. Hỏi n đường thẳng đó
chia mặt phẳng thành bao nhiêu miền.
Chứng minh. Với n = 1 chia mặt phẳng thành 2 miền,
Với n = 2 chia mặt phẳng thành 4 miền,
Với n = 3 chia mặt phẳng thành 7 miền.
Gọi Sn là số miền con chia bởi n đường thẳng. Suy ra:
2(2 + 1) 3(3 + 1)
S1 = 2, S2 = 4 = 1 + , S3 = 1 + = 7, . . .
2 2

6
n(n + 1)
Từ đó ta đi tới chứng minh bằng quy nạp: Sn = 1 + .
2
k(k + 1)
Giả sử đúng với n = k . Suy ra Sk = 1 + . Khi đó k + 1 đường thẳng. Vậy
2
đường thẳng k + 1 cắt k đường thẳng trước tại thành k + 1 miền mới. Suy ra:
k(k + 1) (k + 1)(k + 2)
Sk+1 = Sk + (k + 1) = 1 + + (k + 1) = 1 + .
2 2
Vậy đẳng thức đúng.

Ví dụ 1.2.2. Cho n điểm A1 , A2 , . . . , An trên cùng mặt phẳng sao cho không
có 3 điểm nào thẳng hàng, không có 4 điểm nào tạo thành hình bình hành.
Gọi I1 , I2 , . . . , Im là tất cả các trung điểm của các đoạn tạo thành từ các đoạn
Ai Aj (1 ≤ i < j ≤ n). Gọi N là tổng độ dài của mọi đoạn thẳng với hai đầu
mút là Ai Aj . M là tổng độ dài của mọi đoạn thẳng với d là hai đầu mút. Gọi
các đỉnh của đa giác đều đã cho là: Ii , Ij . Chứng minh rằng:
n2 − 3n + 2
M≤ · N.
4
Chứng minh. Ta có M, N, P lần lượt là trung điểm của AB, BC, CA thì ta có:
1
M N + N P + P M = (AB + BC + CA). (1)
2
Với M, N, P, Q, R, S lần lượt là trung điểm của AB, CD, BC, DA, AC, BD thì ta
có:
1
M N + P Q + RS ≤ (AB + BC + CD + DA + AC + BD). (2)
2
Dễ thấy (1) là hiển nhiên vì theo tính chất đường trung bình của tam giác còn
(2.3.1) ta dựa vào nhận xét sau:
1
P Q ≤ P K + KQ = (AB + CD). (3)
2
Do đó dựa vào (1) và (2.3.1) là đúng. Khi đó, với n điểm A1 , A2 , . . . , An ta thiết
lập các tam giác và tứ giác, rồi lập ra mọi đẳng thức (1) và mọi bất đẳng thức
(2.3.1) và tương ứng cộng vế với vế ta thu được bất đẳng thức (3).
Bây giờ với bất đẳng thức (3) ta xét các giá trị của hai vế theo cách khác
nhau là M .
2
Mỗi đoạn Ii Ij thuộc vào n − 2 tam giác và Cn−2 tứ giác nên vế phải (3) bằng:
1 2 n2 − 3n + 2
(n − 2 + Cn−2 )·N = · N.
2 4
n2 − 3n + 2
Như vậy ta sẽ thu được: M ≤ · N . Vậy các đa giác đều được lát
4
mặt phẳng chỉ có tam giác đều, hình vuông và lục giác đều.

7
1.3 Một số ví dụ về bài toán hình học tổ hợp
1.3.1 Các bài toán đếm trong hình học tổ hợp

Trong các bài toán về đếm số đoạn thẳng, số góc, số đa giác, đôi khi ta cần
dùng đến công thức tính số tổ hợp chập k của n phần tử (tức là số tập hợp
con gồm k phần tử trong n phần tử):
n(n − 1)(n − 2) . . . [n − (k − 1)]
Cnk =
k!
n(n − 1) 7·6·5
Chẳng hạn: Cn2 = , C73 = = 35. Ta cũng nhắc lại ở đây công
2 3!
n(n − 3)
thức tính số đường chéo của đa giác lồi n cạnh: .
2
Bài toán 1.3.1. [1] Một n- giác lồi (n > 3) có nhiều nhất bao nhiêu đường
chéo có độ dài lớn nhất?
Lời giải. Ta có một đa giác lồi có AC là đường chéo lớn nhất và có hai cạnh
bằng AC , đó là AB và AD. Ta sẽ chứng minh với mọi đa giác lồi có đường chéo
lớn nhất bằng số cạnh a > 2. Trong số các cạnh đó (các cạnh này không là ba
cạnh của một tam giác), ta chọn được hai cạnh không có đỉnh chung, chẳng
hạn là AB và C D. Ta sẽ chứng minh tồn tại một đường chéo có độ dài lớn hơn
a.
Thật vậy, vì AC + BD > AB + CD = a + a = 2a, nên tồn tại một trong hai
đường chéo AC, BD lớn hơn a, trái với giả thiết đường chéo lớn nhất bằng a.
Vậy trong đa giác lồi, có nhiều nhất là hai cạnh có độ dài bằng đường chéo
lớn nhất.
Bài toán 1.3.2. [1] Kẻ tất cả các đường chéo của một đa giác lồi 7 cạnh. Biết
rằng không có ba đường chéo nào đồng quy. Có bao nhiêu giao điểm của các
đường chéo nằm trong đa giác?
Lời giải. Mỗi giao điểm của hai đường chéo tương ứng duy nhất với một tứ
giác lồi có các đỉnh là các đỉnh của đa giác. Do đó, có bao nhiêu tứ giác lồi thì
có bấy nhiêu giao điểm của các đường chéo nằm trong đa giác. Gọi đa giác lồi
là ABCDEF G, ta có M, N lần lượt là giao điểm của AC với BD và AC . P là
giao điểm của BE và AD. Nên điểm M ứng với tứ giác ABCD, điểm N ứng với
tứ giác ABCE , điểm P ứng với tứ giác ABDE . Số giao điểm phải tìm bằng:
7·6·5·4
C74 = = 35
4!

1.3.2 Các bài toán chứng minh trong hình học tổ hợp

Bài toán 1.3.3. [3] Cho 17 đường thẳng có tính chất: Mỗi đường thẳng cắt
m
hình chữ nhật cho trước thành hai tứ giác có tỉ số diện tích bằng , (m, n > 0).
n

8
Chứng minh rằng ít nhất 5 trong các đường thẳng trên cùng đi qua một điểm.

Lời giải. Nhận xét: Các đường thẳng đã cho không thể cắt hai cạnh kề nhau
của hình chữ nhật, vì cắt hai cạnh kề nhau tạo thành tam giác và ngũ giác
sẽ mâu thuẫn với giả thiết. Suy ra các đường thẳng này phải đi qua hai cạnh
đối diện của hình chữ nhật (không đi qua đỉnh). Giả sử đường thẳng M N cắt
hình chữ nhật tạo thành hai tứ giác là AM N D và M N CN .
Suy ra:
dt(AM N D) m
= .
dt(M BCN ) n
Gọi P và Q là trung điểm của AD và BC , cắt M N tại E . Suy ra:
(AM + DN )AD m PE m
= ⇒ =
(M B + CN )BC n EQ n
Suy ra:
PE m PE m
= ⇒ =
EQ + P E n+m PQ m+n
Suy ra E cố định do tính chất đối xứng của hình chữ nhật. Do đó trên P Q tồn
tại điểm F , đường thẳng nào đi qua F cũng cho kết quả tỉ số diện tích hai tứ
m
giác bằng .
n
Tương tự tồn tại hai điểm G và J trên trục đối xứng HK thỏa mãn bất cứ
đường thẳng nào đi qua G và J đều thỏa mãn giả thiết chia hình chữ nhật
m
thành hai tứ giác có tỉ số diện tích bằng . Từ đó tồn tại bốn điểm E, F, G, J
n
trên P Q và HK thỏa mãn bất cứ đường thẳng nào đi qua cắt hai cạnh đối
m
diện hình chữ nhật thành hai tứ giác thỏa mãn tỉ số diện tích bằng . Như
n
vậy ta đã chỉ ra có “ 17 chú thỏ và 4 cái lồng”, đó là 17 đường thẳng và 4 điểm
17 − 1
E, F, G, J . Theo nguyên lý Dirichlet thì luôn có ít nhất 1 + [ ] = 5 đường
4
thẳng luôn đi qua 1 điểm.

Bài toán 1.3.4. [3] Cho 19 điểm trong đó không có ba điểm nào thẳng hàng,
nằm trong một lục giác đều có cạnh bằng 1. Chứng minh rằng luôn tồn tại
một tam giác (đỉnh là ba trong 19 điểm trên) có ít nhất một góc không lớn
3
hơn 45◦ và nằm trong đường tròn có bán kính nhỏ hơn .
5
Lời giải. Lục giác đều được chia thành 6 tam giác đều có cạnh bằng 1. Vậy ta
19 − 1
có “ 6 lồng và 19 thỏ”. Suy ra luôn có ít nhất 1 + [ ] = 4 điểm nằm trong
6
6 tam giác đó.
Gọi 4 điểm nằm trong tam giác đều có cạnh bằng 1 là A, B, C, D. Vị trí của
A, B, C, D được sắp xếp như sau:

9
• A, B, C, D là tứ giác lồi, suy ra ∠A + ∠B + ∠C + ∠D = 360◦ . Suy ra có ít
nhất một góc ≤ 90◦ , giả sử ∠A ≤ 90◦ ⇒ ∠DAC + ∠CAB ≤ 90◦ . Vậy trong
hai góc ∠DAC, ∠CAB luôn có một góc không lớn hơn 45◦ .

• Có một điểm nằm trong tam giác tạo bởi ba điểm còn lại.
Giả sử điểm D nằm trong tam giác ABC .

- Nếu ∠CDB ≥ 90◦ ⇒ ∠DBC + ∠DCB ≥ 90◦ . Suy ra một trong hai góc
∠DBC, ∠DCB có một góc nhỏ hơn 45◦ ⇒ tam giác thỏa mãn là tam giác
DBC .

- Nếu ∠CDB < 90◦ ⇒ CAB < 90◦ . Suy ra ∠CAD hoặc ∠DAB luôn có một
góc nhỏ hơn 45◦ ⇒ tam giác thỏa mãn là ACD hoặc ADB .

Tam giác đều có cạnh bằng 1 ⇒ bán kính đường tròn ngoại tiếp tam giác là
√ √
3 1 1 1 9 3 3
· 25 < 27 ⇒ < ⇒ < ⇒ < .
3 27 25 3 25 3 5
Bài này đã sử dụng hai lần nguyên lý Dirichlet.

Bài toán 1.3.5. [3] Cho tam giác đều có cạnh bằng 1, lấy 17 điểm tùy ý trong
tam giác đều. Chứng minh rằng luôn tồn tại hai điểm trong số 17 điểm có
1
khoảng cách không vượt quá .
4
1
Lời giải. Chia tam giác đều thành 16 tam giác đều có cạnh bằng , từ 17 điểm
4
đã cho luôn có một tam giác chứa ít nhất 2 điểm (kể cả điểm nằm trên cạnh).
1
Gọi hai điểm bất kì M, N nằm trong tam giác đều ABC có cạnh bằng ,
4
đường thẳng M N cắt hai cạnh tam giác tại P và Q (giả sử hai cạnh AB, AC ).

- Nếu M
\ QA > 90◦ ⇒ tam giác AP Q là tam giác tù. Suy ra
1
M N < AQ < AB ≤ .
4

- Nếu M
\ QA < 90◦ ⇒ P[CQ = 180◦ − M
\ QA ≤ 90◦ .
Suy ra P QC là tam giác tù ⇒ M N ≤ P Q < P C ≤ BC .

Vậy cả hai trường hợp ta luôn có BC > M N.

Bài toán 1.3.6. [3] Cho điểm P nằm trong đa giác 2n cạnh, kẻ các đường
thẳng qua P và mỗi đỉnh đa giác. Chứng minh rằng bao giờ cũng tìm được
một cạnh của đa giác sao cho không có một đường thẳng nào nói trên cắt cạnh
này.

10
Lời giải. Theo giả thiết cho đa giác có 2n cạnh. Suy ra đa giác có 2n đỉnh gọi
đa giác đó là A1 A2 . . . Ak . . . A2n−1 A2n .
Giả sử P nằm trên đường chéo nào đó chẳng hạn đường chéo A1 Ak . Suy ra
đường thẳng P A1 và P Ak trùng nhau. Như vậy, 2n − 2 đường thẳng còn lại chỉ
cắt phần trong tối đa của đa giác là 2n − 2 cạnh. Suy ra có ít nhất một cạnh
của đa giác thỏa mãn bài toán.
Điểm P không nằm trên bất cứ đường chéo nào của đa giác A1 A2 . . . Ak . . . A2n−1 A2n .
Nối An với An+1 suy ra đường thẳng An An+1 chia đa giác A1 A2 . . . Ak A2n−1 A2n
thành hai đa giác, mỗi đa giác có n cạnh, điểm P nằm ở một trong hai đa
giác đó. Giả sử P nằm trong đa giác A1 A2 . . . An+1 . Từ P nối với các đỉnh
A1 , A2 , . . . , Ak , . . . , A2n−1 , A2n .
Vậy suy ra có n + 1 đường thẳng đó là P An+1 , P An+2 , . . . , P A1 không thể cắt
các cạnh An+1 An+2 , An+2 An+3 , . . . , A2n A1 thỏa mãn bài toán.

Bài toán 1.3.7. [3] Cho tứ giác ABCD. Chứng minh rằng bốn đường tròn có
đường kính là bốn cạnh của tứ giác phủ kín miền tứ giác ABCD.

Lời giải. Giả sử M là điểm trong tứ giác ABCD. Suy ra

A
\ MB + B
\ MC + C
\ MD + D
\ M A = 360◦ .

Không mất tính tổng quát, giả sử A \M B là góc lớn nhất trong bốn góc
A
\ M B, B
\ M C, C
\ M D, D
\ M A. Theo định lý Dirichlet suy ra A
\ M B ≥ 90◦ ⇒ M
nằm trong đường tròn đường kính AB .
Vậy bốn đường tròn có đường kính AB, BC, CD, DA phủ kín miền tứ giác
ABCD.

Bài toán 1.3.8. [3] Trong mặt phẳng có năm điểm không có ba điểm nào
thẳng hàng. Chứng minh rằng tồn tại ít nhất một tứ giác lồi có đỉnh là trong
các điểm đã cho.

Lời giải. Trước hết ta dựng bao lồi của năm điểm đã cho:

(1) Nếu bao lồi là tứ giác, bài toán đã được chứng minh;

(2) Nếu bao lồi là tam giác. Giả sử tam giác đó là ABC , khi đó điểm D và E
nằm trong tam giác (D và E không nằm trên các cạnh của tam giác ABC
vì không có ba điểm nào thẳng hàng).

Suy ra điểm D nằm trong ba tam giác EAB, EBC, ECA. Giả sử D nằm trong
tam giác EBC suy ra B hoặc C và D cùng thuộc nửa mặt phẳng bờ AE .
Nếu là B thì tứ giác AEDB là lồi.
Nếu là C thì tứ giác AEDC là tứ lồi cần chỉ ra.

11
Bài toán 1.3.9. [3] Có thể dùng ba hình vuông có cạnh bằng 4, phủ kín một
hình vuông có cạnh bằng 5 hay không (với các hình vuông cạnh 4 không có
phần chung nhau)?
Lời giải. Gọi hình vuông có cạnh bằng 5 là ABCD. Giả sử có ba hình vuông
cạnh bằng 4 phủ kín hình vuông ABCD (ba hình vuông có cạnh bằng 4 không

có phần chung nhau). Độ dài đường chéo hình vuông cạnh 4 bằng 4 2 > 5.
Theo Định lý Dirichlet suy ra bốn đỉnh A, B, C, D nằm trong ba hình vuông
cạnh 4. Vậy suy ra có một hình vuông cạnh 4 chứa hai đỉnh vuông ABCD. Do
đó đỉnh giao của ba hình vuông cạnh bằng 4 nằm trong hình vuông ABCD.
Hạ AH⊥M N ⇒ AH ≤ 4.
Gọi M và N là giao điểm hình vuông ABCD với hai cạnh hình vuông cạnh
4, AM, AN là cạnh huyền của tam giác ABM và tam giác ADN .
Suy ra AM, AN ≥ 5. Áp dụng Định lý Pythagoras suy ra
AM 2 = AH 2 + HM 2 ⇒ HM ≥ 3.

Tương tự đối với tam giác vuông AHN ⇒ HN ≥ 3 ⇒ HM + HN ≥ 6. Mâu


thuẫn. Vậy ABCD không thể nằm trọn trong ba hình vuông.
Bài toán 1.3.10. [3] Cho tam giác đều ABC , các điểm D, E, I lấn lượt nằm
trên các cạnh BC, CA, AB thỏa mãn BAD
[ + CBE[ + ACI d = 120◦ . Chứng minh
rằng các tam giác: BAD, CBE, ACI phủ kín tam giác đều ABC .
Lời giải. Giả sử P là điểm nằm trong tam giác ABC mà các tam giác
BAD, CBE, ACI không phủ điểm P .
Từ đó suy ra
BAD
[ < BAP
[ , CBE
[ < ACI [ ⇒ BAD+
d < ACP [ CBE[ + ACI
d = BAP
[ + CBP
[ + ACP
[.

Theo giả thiết BAD


[ + CBE
[ + ACId = 120◦ ⇒ BAP [ + CBP [ > 120◦ .
[ + ACP
Nếu P trùng với tâm đường tròn ngoại tiếp tam giác ABC thì suy ra
BAD
[ + CBE d = 30◦ + 30◦ + 30◦ = 90◦ .
[ + ACI

Mâu thuẫn với giả thiết BAD


[ + CBE[ + ACId = 120◦ . Suy ra P khác O. Từ đó
suy ra P A, P B, P C không bằng nhau.
Kéo dài P A, P B, P C cắt đường tròn ngoại tiếp tam giác ABC tại K, M, N .
Suy ra
BAP
[ + CBP
[ + ACP
[ =K\MP + C
\ KM + AKN
[ <M\KB + C
\ KM + AKN
[ < BKC.
[

Bài toán 1.3.11. [3] Chứng minh rằng nếu đi dọc theo các cạnh của các ô
vuông cơ sở từ một đỉnh bất kì rồi trở về đỉnh ban đầu sau hữu hạn bước (có
độ dài bằng cạnh ô vuông cơ sở), thì số bước đi của ta sẽ là số chẵn.

12
Lời giải. Ta tưởng tượng rằng các ô vuông được kẻ sao cho cạnh của nó nằm
theo phương nằm ngang và thẳng đứng. Để trở về đỉnh ban đầu, ta đi lên trên
bao nhiêu bước thì đi xuống dưới bấy nhiêu bước, đi sang trái bao nhiêu bước
thì phải đi về bên phải bấy nhiêu bước. Do đó tổng số bước đi phải là số chẵn.

Bài toán 1.3.12. [3] Cho một đa giác lồi. Chứng minh rằng tồn tại một hình
bình hành có diện tích không quá hai lần diện tích đa giác sao cho các đỉnh đa
giác nằm trong hoặc trên cạnh của hình bình hành.

Lời giải. Gọi d là đường thẳng chứa cạnh CD của đa giác, A là điểm nằm
trên đa giác và có khoảng cách đến đường thẳng d là lớn nhất, qua A kẻ đường
thẳng song song với CD.
Gọi B và E là hai đỉnh có khoảng cách đến AD là lớn nhất về hai phía AD.
Qua B và E kẻ đường thẳng song song với AD, các đường thẳng này tạo thành
hình bình hành P QM N . Suy ra các đỉnh của đa giác nằm trong hoặc trên cạnh
của hình bình hành P QM N .
Suy ra SAED + SABD ≤ SABCDEF (dấu bằng xảy ra khi đa giác là tứ giác). Mà
1 1
SAED + SABD = SP QM N ⇒ SP QM N ≤ SABCDEF ⇒ SP QM N ≤ 2SABCDEF .
2 2
Bài toán 1.3.13. [3] Mỗi một điểm trong mặt phẳng được tô màu bởi hai
màu xanh hoặc đỏ. Chứng minh rằng luôn tồn tại một tam giác đều có ba đỉnh
cùng màu.

Lời giải. Gọi tam giác đều đó là ABC . Ba điểm A, B, C không cùng màu.
Giả sử A, B cùng màu đỏ và C màu xanh.
Kéo dài AB, AC sao cho BE = AB, CI = AC . Suy ra tam giác AEI đều.
Gọi D là trung điểm EI kéo dài DB, DC sao cho BM = BD, CN = CD.
Suy ra tam giác DM N là tam giác đều và M, N, A thẳng hàng. Do đó các tam
giác CM E, ABM, BED, CDI là tam giác đều.

• Tam giác ABM có A, B đỏ suy ra M xanh.

• Tam giác CM E có C, M xanh suy ra E đỏ.

• Tam giác BED có B, E đỏ suy ra D xanh.

Xét tam giác CDI :

• Nếu I màu xanh suy ra tam giác CDI có ba đỉnh cùng màu.

• Nếu I đỏ suy ra tam giác AEI có ba đỉnh màu đỏ.

Vậy ta luôn tồn tại tam giác CDI hoặc AEI cùng có ba đỉnh cùng màu.

13
Bài toán 1.3.14. [3] Mỗi điểm trên mặt phẳng được tô bằng mày đen hoặc
đỏ. Chứng minh rằng ta có thể tìm được ba điểm cùng màu mà mỗi cặp điểm
có khoảng cách bằng một hoặc có thể tìm được ba điểm cùng màu có khoảng

cách bằng 3 bởi hai màu xanh hoặc đỏ. Chứng minh rằng luôn tồn tại một
tam giác đều có ba đỉnh cùng màu.
Lời giải. Giả sử không tồn tại 3 điểm cùng màu mà mỗi cặp điểm cách nhau
bằng 1. Ta sẽ chứng minh rằng ta có thể tìm được ba điểm cùng màu và cách

nhau bằng 3.
Theo giả thiết, suy ra tồn tại hai điểm khác nhau mà khoảng cách bằng 1, ta
lấy hai điểm A, B khác có khoảng cách bằng 2.
Gọi I là trung điểm AB , IA = IB = 1 và điểm I cùng màu với A hoặc B .
Giả sử I và B cùng màu đỏ, suy ra A màu đen, dựng hai tam giác đều IBC
và IBD. Do đó C và D đối xứng nhau qua AB , hai tam giác đều này có cạnh
bằng 1.
Theo giả thiết trên không tồn tại 3 điểm cùng màu, mỗi cặp điểm cách nhau
bằng
• D và C màu đen,

• A, D, C màu đen.

DC cắt AB tại H suy ra DH vuông góc AB và HI = HB . Suy ra



3 √
DH = ⇒ DH = 3.
2
Tam giác DAB có IA = IB = ID = 1.
[ = 90◦ , DAB
Vậy suy ra ADB [ = 30◦ , DAC
[ = 60◦ . Do đó tam giác ADC là tam

giác đều có cạnh bằng 3. Ba điểm A, D, C cùng màu đen và cách nhau một

khoảng bằng 3.
Bài toán 1.3.15. [3] Bàn cờ vua 8 × 8, các ô được tô trắng đen xen kẽ nhau.
Ở mỗi bước xét một hàng hoặc cột, sau đó trong hàng (hoặc cột) chọn ra tay
thay đổi tất các các ô trong hàng (hoặc cột) theo quy tắc: ô đen tô thành ô
trắng và ngược lại. Hỏi bằng cách ấy có thể đến một lúc nào đó trên bàn cờ
chỉ có duy nhất một ô đen được không? Chứng minh.
Lời giải. Giả sử trước khi tô lại một hàng (hoặc một cột) chọn ra, có k ô đen
và 8 − k ô trắng. Sau khi tô lại hàng (hoặc cột) sẽ có k ô trắng và 8 − k ô đen.
Sau một lần đổi số ô đen là: (8 − k) − k = 8 − 2k = 2(4 − k).
Thay đổi một số chẵn ô đen. Như vậy tính chẵn, lẻ của ô đen thay đổi suốt
quá trình ta tô màu, bàn cờ lúc đầu có 32 ô đen.
Không lúc nào nhận được trên bàn cờ chỉ có một ô đen.

14
Bài toán 1.3.16. [3] Trong mặt phẳng tọa độ xOy , có tồn tại đa giác đều 45
cạnh mà các đỉnh của nó là các số hữu tỉ không?

Lời giải. Đa giác đều A1 A2 A3 . . . A45 , giả sử tam giác A1 A16 A31 là tam giác
đều có các đỉnh là các số hữu tỉ bao gồm tam giác A1 A16 A31 bằng hình vuông
ABCD. Tọa độ của A1 A16 A31 là các số hữu tỉ.
Suy ra tọa độ A, B, C, D cũng là các số hữu tỉ.
Do đó diện tích hình vuông ABCD và các tam giác AA1 A16 , BA16 A31 , DA1 A31
là các số hữu tỉ. Suy ra

SA1 A16 A31 = SABCD − SAA1 A16 − SBA16 A31 − SDA1 A31

là các số hữu tỉ.


3
Mặt khác SA1 A16 A31 = A1 A16 là số vô tỉ. Vậy điều đó không xảy ra tọa độ các
4
đỉnh là số hữu tỉ.

Bài toán 1.3.17. [3] Cho đa giác ABCDEGHK có các góc ở đỉnh bằng nhau,
độ dài các cạnh là số nguyên. Chứng minh cặp cạnh đối song song và bằng
nhau.

Lời giải. Theo giả thiết các góc có đỉnh bằng nhau, theo công thức tính tổng
các đa giác lồi. Suy ra mỗi góc ở đỉnh đa giác lồi có giá trị là:
(n − 2)180◦ 6 · 180◦
= = 135◦ .
n 8
Kéo dài cạnh của đa giác ABCDEGHK cắt nhau thành tứ giác P QM N và
IJSR. Suy ra
AK = 180◦ − KAB
P[ [ = 180◦ − 135◦ = 45◦ .

Tương tự các góc P[KA = 45◦ ⇒ tam giác P AK là tam giác vuông cân. Suy ra
tứ giác P QM N là hình chữ nhật. Do đó AB song song EG và CD song song
với HK .
Chứng minh tương tự IJRS cũng là hình chữ nhật. Suy ra BC song song
GH, DE song song với AK .
Tam giác P AK là tam giác vuông cân. Suy ra
AK BC ED GH
AP = √ , BQ = √ , M E = √ , N G = √ ,
2 2 2 2
AK BC
P Q = P A + AB + BQ = √ + AB + √ ,
2 2
DE GH
M N = M E + EG + GN = √ + EG + √ .
2 2

15
Có P QM N là hình chữ nhật suy ra P Q = M N . Suy ra
AK BC DE GH
√ + AB + √ = √ + EG + √
2 2 2 2
1
√ (AK + BC − DE − GH) = EG − AB.
2
Giả thiết độ dài các cạnh là số nguyên. Suy ra AK +BC +DE −GH và EG−AB
là các số nguyên. Do đó vế trái là số vô tỉ, vế phải là số nguyên.
Suy ra EG − AB = 0 ⇒ EG = AB.
Chứng minh tương tự CD = HK và AK = DE, BC = GH .
Bài toán 1.3.18. [3] Cho hai hình vuông ABCD, P QM N có cạnh bằng nhau,
mỗi cạnh hình vuông được tô màu giống nhau xanh hoặc đỏ, hai hình này xếp
trồng lên nhau tạo thành bát giác. Chứng minh rằng bát giác có tổng độ dài
bốn cạnh màu đỏ bằng tổng độ dài bốn cạnh màu xanh.
Lời giải. Xét bát giác AP N QCM DN nhận thấy:

SABCD +SP AB +SQBC +SM CD +SN DA = SP QM N +SAP N +SBP Q +SCQM +SDM N .

Gọi khoảng cách từ A, B, C, D đến các cạnh N P, P Q, QM, M N theo thứ tự h1,

h2, h4 và khoảng cách từ P, Q, M, N đếm các cạnh AB, BC, CD, DA theo thứ tự
là h5, h6, h7, h8. Hai hình vuông có cạnh bằng nhau suy ra SABCD = SP QM N .
Các tam giác AA1 A8 , P A1 A2 , BA2 A3 , QA3 A4 , CA4 A5 , M A5 A6 , DA6 A7 , N A7 A8 là
các tam giác vuông có

AA
\ 1 A8 = P\
A1 A2 = QA
\ 3 A4 = M
\ A5 A6 = N\
A7 A8 .

Suy ra các tam giác đồng dạng


h1 h2 h3 h4 h5 h6 h7 h8
= = = = = = = = k.
A1 A8 A2 A3 A4 A5 A6 A7 A1 A2 A3 A4 A5 A6 A7 A8
Suy ra

A1 A8 + A2 A3 + A4 A5 + A6 A7 = A1 A2 = A3 A4 = A5 A6 + A7 A8 .

Bài toán 1.3.19. [3] Chứng minh rằng nếu ba đỉnh của một hình bình hành
là các điểm nguyên. Chứng minh rằng đỉnh thứ tư cũng là điểm nguyên.

16
Lời giải. Giả sử hình bình hành ABCD có ba đỉnh A, B, C là các điểm nguyên.
Giả sử tọa độ của A(x1 , y1 ); B(x2 , y2 ); C(x3 , y3 ) với x1 , x2 , x3 , y1 , y2 , y3 là các số
nguyên, gọi giao điểm hai đường chéo là I, ABCD là hình bình hành.
Gọi M, Q, J, N, P lần lượt là chân đường vuông góc hạ từ các điểm A, D, I, B, C
xuống trục tung, ta có: AM + CP = 2IJ = BN + DQ. Suy ra:

x + 1 + x3 = x2 + x4

x 4 = x1 + x3 − x2

Suy ra vế phải là số nguyên. Vậy x4 là số nguyên.


Chứng minh tương tự được y4 là số nguyên.

Bài toán 1.3.20. [3] Cho tam giác ABC mà các đỉnh của nó là các điểm
nguyên. Chứng minh rằng 2SABC là số nguyên.

Lời giải. Ba điểm A, B, C là các điểm nguyên, dựng hình bình hành ABCD.
Theo bài 1.3.19, đỉnh D là điểm nguyên. Suy ra SABCD = 2SABC .
Qua D và B dựng hình chữ nhật BEDI sao cho các cạnh song song với trục
tung và trục hoành. Do đó các đỉnh I và E là các điểm nguyên. Suy ra:

SABCD = SBEDI − SIQAP − SAQD − SAP B − SCN EM − SBCN − SCM D .

Mặt khác, tam giác AQD = CN B ; tam giác AP B = CM D. Suy ra

SAQD = SBCN , SAP B = SCM D ,


SIQAP − SAQD − SAP B − SCN EM − SBCN − SCM D = SIQAP − 2SAQD − 2SAP B − SCN EM
= IQ · QA + AQ · QD + AP · P B + N C · CM
là số nguyên. Suy ra SABCD = 2SABC là số nguyên.

Bài toán 1.3.21. [3] Cho ngũ giác có tất cả các cạnh bằng a. Chứng minh
rằng có thể đặt trong đó một tam giác đều cạnh a.

Lời giải. Giả sử đường chéo AC lớn nhất của ngũ giác ABCDE có các cạnh
bằng a. Ta sẽ chứng minh có tam giác đều cạnh a nằm trong tứ giác ACDE .
Trước hết ta chứng minh hai mệnh đề

17
(1) Chứng minh AEC[ > 60◦ . Ta có ngũ giác ABCDE có tổng các góc bằng
[ ≥ 1 · 540◦ = 180◦ , các cạnh
540◦ , AC là đường chéo lớn nhất. Suy ra ABC
5
ngũ giác bằng a. Suy ra AC > AB = a.
Theo giả thiết, suy ra AC ≥ AE, AC ≥ AD. Suy ra AC là cạnh lớn nhất
của tam giác ACE và ACD. Do đó hai tam giác này không phải tam giác
[ > 60◦ , ADC
đều. Vậy AEC [ > 60◦ .

[ ≥ 60◦ hoặc
(2) Ta chứng minh một trong các bất đẳng thức sau là đúng: EAC
[ ≥ 60◦ .
ACD
[ < 60◦ , ACD
Giả sử EAC [ < 60◦ , CD và AE cắt nhau tại F . Tam giác AF C
ta có AC > AF và AC > F C .
Suy ra AF
[ C > 60◦ , do AC < 2a ⇒ AF < 2a, F C < 2a.
1
Gọi M, N là trung điểm của F A, F C , suy ra M N = AC < a.
2
Tương tự AM < a, CN < a. Mặt khác AE = a, CD = a. Do đó E ở giữa
F và M , và D ở giữa F và N . Suy ra A = ED ≤ M N < a (vô lý). Suy ra
[ ≥ 60◦ hoặc ACD
EAC [ ≥ 60◦ .
[ ≥ 60◦ . Trong tam giác AEC những góc tại đỉnh A và E không
Giả sử EAC
nhỏ hơn 60◦ , AE = a.
Vậy suy ra trong tam giác này dựng được tam giác đều cạnh a.

Bài toán 1.3.22. [3] Chứng minh rằng với mọi số tự nhiên n ≤ 4, khẳng định
sau đây đúng: Mỗi tứ giác nội tiếp chia thành n tứ giác mà mỗi tứ giác này
cũng là tứ giác nội tiếp đường tròn.
Lời giải. Tứ giác ABCD là tứ giác nội tiếp. Giả sử Ab là góc nhỏ nhất của tứ
giác ABCD. Ta lấy điểm A1 tổng góc DAB , từ A1 kẻ các đường song song với
AB và AD cắt cạnh BC, CD tại B1 , D1 .
Trên cạnh AB và AD lấy hai điểm I và E sao cho A[ 1 IB = B,
b A\ b.
1 ED = D

⇒ Tứ giác A1 B1 BI và A1 D1 DE là hình thang cân.

⇒ Tứ giác A1 B1 BI và A1 D1 DE luôn nội tiếp đường tròn.

18
ABCD là tứ giác nội tiếp, suy ra B b = 180◦ , AEI
b+D [1 = 180◦ . Suy ra
d + AIA
AEA1 I là tứ giác nội tiếp.
Tương tự tứ giác A1 B1 CD1 là tứ giác nội tiếp.
Như vậy n = 4, các tứ giác A1 B1 BI và A1 D1 DE là hình thang cân ta có thể
chia thành nhiều tứ giác nội tiếp.
Bài toán 1.3.23. [3] Chia một tam giác đều thành n2 tam giác đều bằng
nhau. Số tam giác đó được đánh số từ 1, 2, . . . , m sao cho các tam giác với các
số liên tiếp phải có cạnh chung. Chứng minh rằng m ≤ n2 − n + 1.
Lời giải. Chia các cạnh của tam giác đều đã cho thành n phần bằng nhau, từ
các điểm chia ta kẻ các đường thẳng song song với các cạnh, cắt tam giác tạo
thành n2 tam giác đều bằng nhau.
Khi đó tổng các tam giác đều được tạo thành (theo hàng) là:

1 + 3 + 5 + · · · + (2n − 1) = n2 .

Ta tô màu các tam giác đều nhỏ bằng hai màu đen trắng xen kẽ nhau, tổng
các ô trắng (theo hàng) là:
n(n − 1)
1 + 2 + · · · + (n − 1) = .
2
Theo các đánh số tam giác, hai tam giác thứ tự phải có cạnh chung.
⇒ Hai tam giác này màu khác nhau.

⇒ Số tam giác ô đen chỉ hơn tam giác ô trắng 1.


n(n − 1) n(n − 1)
m≤ + + 1 = n2 − n + 1.
2 2
Bài toán 1.3.24. [3] Chứng minh rằng số đường chéo đa giác lồi n cạnh (n ≤ 4)
n(n − 3)
bằng .
2
Lời giải. Gọi Sn là tổng các đường chéo đa giác lồi n cạnh.
Với n = 4 tứ giác có hai đường chéo (S4 = 2). Giả sử đúng với n = k (với đa
k(k − 3)
giác lồi k cạnh) đa giác lồi có Sk = đường chéo. Ta chứng minh đúng
4
với đa giác lồi có n = k + 1 cạnh. Khi đó sẽ thêm k − 2 đường chéo, đó là các
đường nối đỉnh Sk+1 với các đỉnh A2 , A3 , . . . , Ak−1 . Suy ra
k(k − 3) k2 − k − 2 (k + 1)(k − 2)
Sk = + (k − 2) + 1 = = .
4 2 2
Bài toán 1.3.25. (IMO lần thứ 14-1972) Cho n > 4. Chứng minh rằng:
mọi tứ giác nội tiếp đường tròn đều có thể chia thành n tứ giác nội tiếp đường
tròn.

19
Lời giải. Ta có thể phân chia thành 4 tứ giác nội tiếp. Thật vậy, ta lấy P tùy ý
bên trong tứ giác ABCD. Gọi K là một điểm trên cạnh AB , nối P K . Tiếp đên,
ta lấy trên cạnh BC một điểm L sao cho góc KP [L = 180◦ − B b (điều này chứng
tỏ tứ giác KP LB nội tiếp). Ta lại lấy M trên CD sao cho góc LP[ M = 180◦ − Cb.
Sau đó lấy N trên AD sao cho M \ P N = 180◦ − D
b . Lúc đó, dễ dàng chứng minh
được rằng N\ P K = 180◦ − A
b.
Tuy nhiên, ta còn phải chứng minh rằng các điểm P, L, M lấy như trên thỏa
mãn ý định của chúng ta, nghĩa là nó phải nằm trên các cạnh như đã nói. Điều
này tùy thuộc vào cách lấy điểm K và P mà phần trình bày sau sẽ rõ.
Rõ ràng là nếu tứ giác nội tiếp ABCD có hai cạnh song song với nhau (AB
song song CD) thì bài toán xem như được giải quyết, bởi vì lúc đó, ta có thể
chia nhỏ tứ giác ABCD thành tuỳ ý tứ giác (cũng nội tiếp) bằng n đường thẳng
song song AB song song CD. Vì vậy, bài toán sẽ được giải xong nếu ta chọn
các điểm K và P thế nào cho một trong các tứ giác nội tiếp mới thành lập có
hai cạnh song song nhau. Điều này thì dễ, bởi K tuỳ ý. Do đó, ta lấy P K song
song AD, lúc này, chắc chắn P L song song CD vì: KP [L = 180◦ − B b=D b.
Vấn đề còn lại là xem xét sao cho các điểm K, L, M, N tương ứng nằm trên
các cạnh AB, BC, CD, AD.
Trước hết, xét K và L. K không thể nằm trên AD vì ta có P K song song AD.
Như thế, chỉ cần lấy P đủ gần điểm D thì K sẽ nằm trên cạnh AB (tránh tình
trạng đường P K cắt BC ). Tương tự như thế, ta cũng lấy P đủ gần D để bảo
đảm cho L nằm trên cạnh BC . Lúc đó, ta giả sử cả M lẫn N đều nằm trên
AD, thế thì bằng cách giữ cho K cố định, ta kéo P tiến về gần sát với CD, N
sẽ dịch chuyển lên CD, để lại M nằm trên AD.

Bài toán 1.3.26. (IMO lần thứ 25-1984) Cho d là tổng chiều dài của tất
cả các đường chéo của một đa giác lồi với số đỉnh là n > 3. Gọi p là chu vi của
đa giác đó. Chứng minh rằng:
2d n n+1
n−3< < [ ][ ] − 2.
p 2 2

Lời giải. Xét đường chéo AX , ta gọi B là đỉnh kế tiếp của đa giác kể từ A,
ngược chiều kim đồng hồ và Y là đỉnh kế tiếp của đa giác kế từ X , ngược chiều
kim đồng hồ. Khi đó, ta gọi K là giao điểm của AX và BY . Vì AK + KB > AB
và X + KY > XY nên

AX + BY > AB + XY (1)

Giữ điểm A cố định, cho điểm X thay đổi qua các đỉnh, ta thấy (n − 3) trường
hợp xảy ra, nghĩa là ta thiết lập được tất cả (n − 3) bất đẳng thức như (1).

20
Trong quá trình thiết lập các bất đẳng thức này, ở về trái, mỗi đường chéo
xuất hiện 4 lần, còn ở về phải, mỗi cạnh xuất hiện 2(n − 3) lần. Do đó, khi lấy
tổng các bất đẳng thức này về theo về ta được:
2d
4d > 2(n − 3)p ⇔ n − 3 < .
p

Gọi d, là tổng các đường chéo nối hai đầu mút của đường gấp khúc gồm k cạnh
n
liên tiếp của đa giác, ta có: dk < kp, với k = 2, 3, . . . , ([ ] − 1);
2
n
• d n < [ ] · p khi n lẻ.
[ ] 2
2
n p
• d n < [ ] · khi n chẵn.
[ ] 2 2
2
Vì vậy khi n lẻ ta có:
d = d2 + d3 + · · · + d n
[ ]
2
n
< p(2 + 3 + · · · + [ ])
2
1 n n
= + (2 + [ ])([ ] − 1)
2 2 2
p n n
= ([ ])([ ] + 1) − 2
2 2 2
p n −(n + 1)
= ([ ])[ ] − 2.
2 2 2
Khi n chẵn ta có:
d = d2 + d3 + · · · + d n
[ ]
2
n p n
< p(2 + 3 + · · · + [ ] − 1) + ([ ])
2 2 2
1 n n
= + (2 + [ ])([ ] − 1)
2 2 2
p n2
= ([ ] − 2)
2 2
p n −(n + 1)
= ([ ])[ ] − 2.
2 2 2

21
Chương 2

Một số bài toán về cực trị trong


hình học tổ hợp

Với những kiến thức đã được viết ở Chương 1 và duyệt lại các đề thi học
sinh giỏi, kì thi vào trường chuyên cũng như các kì thi Olympic toán trong
nước và quốc tế. Chương này đưa ra các bài toán cực trị hình học tổ hợp cụ
thể, đã được sắp xếp theo từng dạng: Tìm giá trị lớn nhất; Tìm giá trị nhỏ
nhất;...

2.1 Bài toán về tìm giá trị lớn nhất


Bài toán 2.1.1. (China TST 2007) Cho 63 điểm nằm trên đường tròn bán
kính 10. Đặt S là số các tam giác mà các cạnh của nó lớn hơn 9 còn các đỉnh
của nó thì thuộc 63 điểm đã cho. Tìm giá trị lớn nhất của S ?
Lời giải. Đầu tiên ta chứng minh ba Bổ đề:
Bổ đề 1: Gọi O là tâm đường tròn C ; an là độ dài cạnh của đa giác đều
A1 A2 . . . An . Ta gọi AP B là cung của P nếu thỏa mãn đồng thời: P là điểm
thuộc AP B và AOB [ = 4π . Với n điểm trên đườn tròn C ; có một điểm P thỏa
7
(n + 5)
mãn có [ ] điểm n đã cho là cung của P .
6
Chứng minh: Cho A là một điểm trong n điểm và cung của A là A1 AA6 .
Giả sử rằng: A2 ; A3 ; A4 ; A5 nằm trên cung A1 A6 không chứa A.

Từ A1 A2 = A2 A3 = · · · = A5 A6 ⇒ (Ai OAi+1 ) = với i = 1, 5. Với mỗi điểm Pi
7
(chọn n điểm) trên cung Ai Ai+1 thì tất cả các điểm trên Ai Ai+1 đều được coi
là cung của Pi . Với n điểm đã cho mà có 6 cung của Pi (bao gồm cả cung của
(n − 1) (n + 5)
A). Theo nguyên lý Dirichlet, sẽ có [ ]+1 = [ ] điểm là cung của
6 6
P với P là một trong n điểm đã cho.
Bổ đề 2: Trên cung A1 BA6 của đường tròn C bán kính 10 và có độ dài A1 BA6
5
bằng chu vi của đường tròn; cho 5m + r (0 ≤ r ≤ 5) điểm bất kì. Khi đó ta có
7
số đoạn thẳng nối hai điểm trong các điểm này mà có độ dài lớn hơn 9; nhiều

22
là:
1
10m2 + 4rm + r(r − 1).
2
Chứng minh: Dễ dàng thấy cung A1 BA6 chứa đúng 5 cung Ai Ai+1 với i = 1, 5
sẽ có độ dài không vượt quá a7 < 9. Giả sử rằng có mi điểm trên Ai Ai+1 với
P
i = 1, 5. Khi đó, số đoạn thẳng thỏa mãn lớn nhất là: 1≤i≤j≤5 (mi mj ) với
m1 + m2 + · · · + m5 = 5m + r. Do số các bộ nguyên không âm (m1 + m2 + · · · + m5 )
là hữu hạn (vì tổng luôn bằng 5m + r) nên giá trị lớn nhất của 1 là tồn tại.
Ta chứng minh: Khi 1 đạt giá trị lớn nhất thì:

| mi − mj |≤ 1 (1 ≤ i ≤ j ≤ 5) (1)

Thật vậy: phản chứng (1); ta có thể giả sử: m1 − m2 ≥ 2. Khi đó, ta xét bộ
0 0 0 0 0 0
(m1 ; m2 ; . . . ; m5 ) có: m1 = m1 − 1; m2 = m2 + 1; mi = mi với (i = 3, 4, 5).
0 0 0
Khi đó, có: (m1 ; m2 ; . . . ; m5 ) = 5m + r. Ngoài ra ta có:
0 0 0 0 0
I − 1 = (m1 · m2 − m1 · m2 ) + [(m1 + m2 ) − (m1 + m2 )](m3 + m4 + m5 )
0 0
⇒ I − 1 = m1 − m2 − 1 ≥ 1 ⇒ I ≥ I + 1.

Điều này mâu thuẫn với tính lớn nhất của I , do đó (1) đúng. Từ (1) và từ
0 0 0
(m1 ; m2 ; . . . ; m5 ) = 5m + r ta suy ra: khi I đạt giá trị lớn nhất của các đoạn
thẳng thỏa mãn là:
1
Cr2 (m + 1)2 + Cr1 · C5−r
1 2
· (m + 1) · m + C5−r · m2 = 10m2 + 4rm + r(r − 1).
2
Bổ đề 3: Tập M là tập n điểm bất kì nằm trên đường tròn C bán kính 10 bởi
n = 6m + r (0 ≤ r ≤ 6). Gọi Sn là số các tam giác có 3 đỉnh thuộc M mà độ dài
mỗi cạnh lớn hơn 9. Khi đó, ta có:
1
Sn ≤ 20m3 + 10rm2 + 2r(r − 1)m + r(r − 1)(r − 2).
6
Chứng minh: Ta chứng minh Bổ đề 3 bằng quy nạp theo n.

• Với n = 1; 2 có Sn = 0, Bổ đề 3 đúng.

• Giả sử đúng với n = k ; k = 6m + r (0 ≤ r ≤ 6); tức là ta có:


1
Sk ≤ 20m3 + 10rm2 + 2r(r − 1)m + r(r − 1)(r − 2).
6

Theo Bổ đề 1: với n = k + 1 phải có điểm P để có ít nhất


n+5 k+1+5
[ ]=[ ]=m+1
6 6

23
2
điểm nằm trên cung A1 P A6 có độ dài bằng chu vi đường tròn.
7
Khi đó với bất kì điểm X nào trên cung A1 P A6 thì đều có độ dài
XP ≤ A1 P = A6 P = a7 < 9.

Do đó, số các điểm có khoảng cách tới P lớn hơn 9 nhiều nhất là:
(6n + r + 1) − (m + 1) = 5m + r;
5
và chúng đều nằm ở cung A1 BA6 (không chứa P ) và có độ dài bằng chu vi
7
đường tròn (gọi tập các điểm này là N ). Do đó, theo Bổ đề 2 thì ta suy ra số
tam giác có cạnh lớn hơn có các đỉnh thuộc M và có một đỉnh là P chính bằng
số đoạn thẳng nối hai trong các điểm thuộc N mà có độ dài lớn hơn 9 nhiều
nhất là:
1
Sp = 10m2 + 4rm + r(r − 1).
2
2
(Lưu ý rằng với r = 5 thì Sp = 10(m + 1) ; Bổ đề 2 vẫn đúng.)
Bây giờ xét các tập tam giác thỏa mãn và không có đỉnh là P ; số tam giác
tương ứng với số tam giác thỏa mãn với n = k; k = 6m + r (0 ≤ r ≤ 6); (do bỏ
đi điểm P ). Như vậy ta có:
1 1
Sk+1 = Sk +Sp ≤ [20m3 +10rm2 +2r(r−1)m+ r(r−1)(r−2)]+[10m2 +4rm+ r(r−1)],
6 2
1
⇒ Sk+1 ≤ 20m3 + 10(r + 1)m2 + 2r(r − 1)m + 2r(m + 1)m + r(r − 1)(r + 1).
6
(Lưu ý với r = 5 thì n = k + 1 = 6(m + 1) khi đó Sk+1 = 20(m + 1)3 đẳng thức
trên vẫn đúng.)
Bổ đề đúng với n = k + 1. Vậy theo nguyên lý quy nạp thì Bổ đề 3 đúng với
mọi n.
Quay lại bài toán, ta áp dụng Bổ đề 3 với n = 63 = 6.10 + 3 ta suy ra
1
S ≤ 20 · 103 + 10 · 102 + 2 · 3 · 3 · 10 + · 3 · 2 · 1 = 231221.
6
Ta đi xây dựng cấu hình S = 23121.
Gọi A1 A2 . . . A6 là lúc giác đều nội tiếp đường tròn C . Suy ra Ai Ai+ = 10 > 9.
Giả sử rằng ta chọn được điểm Bi thuộc cung Ai Ai+1 sao cho Bi Ai+1 > 9.

Suy ra (BiOAi+1 ) > (A7 = Ai ) do đó suy ra:
7
2π 2π 2π
B
\ i OAi+1 − Bi \
i OAi = A\ OAi + 1 < − < ⇒ Ai Bi < 9.
6 7 7
Trên các cung A1 B1 ; A2 B2 ; A3 B3 mỗi cung chọn 11 điểm; trên các cung A4 B4 ; A5 B5 ; A6 B6
mỗi cung chọn 10 điểm. Khi đó, tập M có 11 · 3 + 10 · 3 = 63 điểm được chọn
trên đường tròn C . Khi đó, số các tam giác thỏa mãn là:
Sn = C33 · 113 + C32 · C31 · 112 · 10 + C31 · C32 · 11 · 102 + C33 · 103 = 23121.

24
Vậy bài toán được chứng minh hoàn toàn. Kết luận S = 23121.
Vậy giá trị lớn nhất của S là 23121.
Bài toán 2.1.2. (Bulagarian TST 2005) Cho M là đa giác lồi n đỉnh (n ≤ 4).
Giả sử rằng n − 3 đường chéo của M được tô màu xanh và n − 3 đường chéo
khác của M được tô màu đỏ sao cho không có hai đường chéo nào cùng màu
cắt nhau tại một điểm trong M . Tìm số giao điểm lớn nhất trong M của hai
đường chéo khác màu.
Lời giải. Với mỗi đường chéo d được tô xanh, ta gọi số giao điểm của các
đường chéo đỏ trên đoạn của đường chéo d là Cd . Ta cần tìm giá trị lớn nhất
của T = Σd Cd với mọi đường chéo xanh.
Xét di ; dj là hai đường chéo xanh và miền nằm giữa hai đường chéo này có m
đỉnh M . Do có n − 3 đường chéo đỏ nên muốn tìm số giao điểm lớn nhất trên
cả hai đường thẳng này thì số đường chéo đỏ cắt cả hai đường này phải nhiều
nhất (hiển nhiên, vì ta giả sử có x đường chéo cắt cả hai đường và y đường chéo
cắt nhiều nhất 1 đường, thì ta có số giao điểm lớn nhất khi 2x + y = x + (n − 3)
lớn nhất, tức là x lớn nhất). Ta có nhiều nhất n − m − 1 đường chéo đỏ cắt cả
hai đường di , dj (do có m đỉnh nằm trong miền giữa 2 đường này); khi đó có
nhiều nhất (n − 3) − (n − m − 1) = m − 2 đường chéo đó nhiều nhất một trong
hai đường chéo di , dj . Do đó số giao điểm lớn nhất là

Cdi + Cdj ≤ 2(n − m − 1) + (m − 2) = 2n − m − 4 (1)

Giả sử các đường chéo xanh là d1 ; d2 ; . . . ; dn−3 và ta có dạng sau:


• d1 và d2 chia đa giác M thành hai tam giác và đa giác n − 2 cạnh.

• d3 và d4 chia đa giác n − 2 cạnh thành hai tam giác và đa giác n − 4 cạnh.

• ...

• d2k−1 và d2k chia đa giác n − 2k + 2 cạnh thành hai tam giác và giác n − 2k
cạnh.
Đặt n − 3 = 2r + e; e ∈ {0, 1}. Sử dụng (1) ta suy ra:

Cd2k−1 + Cd2k ≤ n + 2k − 4 (2)

Riêng với n − 3 ta có Cdn−3 ≤ n − 3 (vì có nhiều nhất n − 3 đường chéo đỏ cắt


đường chéo xanh).
Gọi T là số giao điểm lớn nhất trong M của hai đường chéo khác màu. Do đó,
ta có: r
X X
T = Cd ≤ (n + 2k − 4) + e(n − 3) = 3r2 + e(3r + 1).
d k=1

25
3
• e = 0 ⇒ n − 3 = 2r ⇒ T0 = (n − 3)2 ,
4
3 3
• e = 1 ⇒ n − 4 = 2r ⇒ T0 = (n − 4)2 + (n − 4) + 1.
4 2
3
Gộp hai kết quả ta có: T0 = [ (n − 3)2 ].
4
n
Ta xây dựng cấu hình để T đạt giá trị T0 như sau. Đặt k = [ ] + 1; đa giác M
2
là A1 A2 . . . An .

• Tô màu xanh các đường chéo: A1 Ai với i = 3, k và Ak Aj với k = k + 2, n.

• Tô màu đỏ các đường chéo: A2 Ai với i = k + 1, n và Ak+1 Aj với k = 3, k − 1.


3
Khi đó kiểm tra trực tiếp ta thấy T = [ (n − 3)2 ].
4
3
Kết luận: Tmax = [ (n − 3)2 ].
4
3
Vậy số giao điểm lớn nhất trong M của hai đường chéo khác màu là (n − 3)2 .
4
Bài toán 2.1.3. [8] Trong mặt phẳng, cho tập hợp A gồm 20102 điểm phân
biệt được đánh số từ 1 đến 20102 sao cho ba điểm bất kì nào trong chúng cũng
không thẳng hàng. Một tứ giác (lồi hoặc lõm) được gọi là “đẹp” nếu các đỉnh
của nó thuộc A và được đánh số bằng 4 số thỏa mãn một trong hai điều kiện
sau:

• Đó là 4 số tự nhiên cách nhau 2010 đơn vị,

• Đó là 4 số tự nhiên liên tiếp và nếu trong đó có chứa số chia hết 2010 thì
số đó phải là lớn nhất.

Nếu tất cả các điểm thuộc tập hợp A lại với nhau sao cho điểm nào thuộc A
cũng thuộc đúng một tứ giác. Tìm số lớn nhất tứ giác “đẹp” được tạo thành.

Lời giải. Rõ ràng cách nối các điểm như trên có thể được mô phỏng bằng 4
số nằm trên một viên gạch 1 × 4 như trên. Ta có thể giải chi tiết như sau: Xét
một bảng ô vuông gồm 2010 × 2010 ô vuông con được diễn các số theo thứ tự
từ trên xuống và trái sang như sau: Trước hết, ta sẽ chứng minh rằng không
thể chia tất cả 20102 điểm đã cho thành các tứ giác “đẹp” được. Rõ ràng 4 số
trên các đỉnh của các tứ giác “đẹp” tương ứng với 4 số bị che đi trên bảng ô
vuông khi đặt một mảnh bìa hình chữ nhật kích thước 1 × 4 vào đó. Ta sẽ
chứng minh rằng không thể cho hết toàn bộ bảng ô vuông này bằng các hình
chữ nhật 1 × 4.
Thật vậy, ta tô màu các ô vuông nằm ở cột chẵn và hàng chẵn. Do bảng có
20122
20102 ô vuông nên số ô vuông bị tô màu là: là số lẻ.
4

26
Giả sử ngược lại rằng ta có thể lấp kín các ô vuông bằng các mảnh bìa. Khi
đó, mỗi bìa ta sẽ che đi hoặc hai ô vuông hoặc không có ô vuông nào của bảng
ô vuông, tức là luôn có một số chẵn ô vuông bị che đi; do đó, số ô vuông bị
che đi trên bảng là một số chẵn. Từ đó ta thấy có mâu thuẫn.
Do đó, không thể che hết bảng ô vuông này bằng các hình chữ nhật 1 × 4 được.
Gọi k là số tứ giác đẹp lớn nhất cần tìm thì k ≤ 1010024. Ta sẽ chứng minh
k = 1010024 bằng cách chỉ ra cách dùng các mảnh bìa che kín bảng ô vuông.
Thật vậy, chia bảng ô vuông thành hai phần:
Phần 1 gồm 2008 cột đầu, ta xếp các mảnh bìa theo các hàng, mỗi hàng có
đúng 502 mảnh bìa. Khi đó, ta sẽ có thể che kín hết phần 1 bởi các mảnh bìa.
Phần 2 gồm 2 cột cuối, ta xếp nối tiếp các mảnh bìa từ trên xuống dưới thì
cuối cùng sẽ còn lại một ô vuông 2 × 2 ở góc dưới cùng của bảng.
Như vậy, ta dùng 10120024 mảnh bìa che được tối đa 20102 − 4 ô vuông của
bảng. Từ đó ta thấy số tứ giác “đẹp” lớn nhất cần tìm là k = 1010024.
Vậy số lớn nhất tứ giác “đẹp” được tạo thành là 1010024.
Bài toán 2.1.4. [8] Trong một hình vuông được tạo bởi 2012 × 2012 ô vuông
có chứa những con bọ, trong một ô vuông có nhiều nhất một con bọ. Vào một
thời điểm nào đó, những con bọ bay lên rồi đậu xuống lại vào các ô vuông, mỗi
ô vuông cũng có nhiều nhất một con bọ. Đối với mỗi con bọ, có thể xem đoạn
thẳng có hướng nối tâm của ô vuông lúc đầu và tâm của lúc sau mà nó đậu
lên tạo thành một vecto. Với mỗi số lượng con bọ ban đầu, xét tất cả những
trường hợp có thể xảy ra với các vị trí đầu tiên và cuối cùng của những con
bọ, hãy tìm độ dài lớn nhất của tổng các vecto.
Lời giải. Xét mặt phẳng chứa hình vuông đã cho là mặt phẳng tọa độ Oxy
với O là tâm của hình vuông lớn.
Gọi tập hợp các tâm của tất cả các ô vuông nhỏ là S , tập hợp những điểm
mà những con bọ dừng lại lúc đầu là M1 ⊆ S . Bỏ qua tường hợp một con bọ
ở cùng một nơi lúc đầu và lúc sau. Xét phép đặt tương ứng M1 đến M2 là f .

27
Để đơn giản, với một điểm x ∈ S , ta kí hiệu x vecto có điểm đầu là O, điểm
cuối là x. Ta có tổng các vecto được tính bằng:
X X X
V = = (f (v) − v) = u− v.
v∈M1 u∈M2 v∈M1

Dễ thấy rằng M1 , M2 ⊆ S, | M1 |=| M2 |. Đồng thời, do ta cần tìm già trị lớn
nhất của độ dài của vector V nêu trên nên chỉ cần xét trường hợp M1 ∩ M2 = φ
(bởi vì nếu M1 , M2 có giao khác rỗng thì phần giao của chúng sẽ cho các vecto
có độ dài bằng 0).
Do cách chọn M1 , M2 hữu hạn nên sẽ tồn tại một cặp nào đó mà | V | đạt giá
trị lớn nhất và hiển nhiên giá trị V đó khác 0. Qua 0 kẻ đường thẳng l vuông
góc với V . Ta sẽ chứng minh các Bổ đề sau:
Bổ đề 1: Nếu | V | đạt giá trị cực đại thì đường thẳng l không đi qua kì điểm
nào và M1 , M2 nằm ở hai phía khác nhau so với l.
S
Chứng minh: Đầu tiên ta thấy rằng M1 M2 = S vì nếu không thì do

| S |= 20122
S
là số chẵn nên có ít nhất hai điểm a, b không có trong M1 M2 , có thể giả sử
góc tạo bởi a − b và V không quá 90◦ thì | V + (a − b) |>| V | (nếu góc đó lớn
hơn 90◦ xét b − a). Suy ra nếu thêm a vào M2 , thêm b vào M1 (hoặc ngược lại)
thì | V | tăng lên, mâu thuẫn do | V | đạt giá trị cực đại.
Hơn nữa, M1 = −M2 vì nếu không thì trong M1 , M2 đều có chứa ít nhất một
cặp điểm đối xứng, cụ thể là a, b ∈ S sao cho a, −a ∈ M1 ; b, −b ∈ M2 .
Có thể giả sử a − b và V hợp thành không quá 90◦ thì
X X
| u− v |=| V + 2(a − b) |>| V | .
S S
u∈(M2 \{b}) {a} u∈(M1 \{a}) {b}

Điều này xảy ra cũng mâu thuẫn với tính lớn nhất của | V | hay S là tập hợp
các điểm đối xứng với nhau.
Ta cũng có l không đi qua bất kì điểm nào thuộc S vì nếu không thì do l đi
qua O nên nếu l đi qua một điểm a ∈ M1 thì nó cũng đi qua điểm −a ∈ M2 .
Khi đó ta đổi chỗ thay a vào M2 , thay ˘a thay vào M1 thì tổng vecto là V + 4a.
Chú ý rằng góc hợp bởi a và V là 90◦ nên | V + 4a |>| V |, mâu thuẫn.
Cuối cùng ta phải có M2 là tập hợp các điểm nằm về một phía so với l. Giả
sử đây là phía có chứa điểm cuối của vecto V và M1 nằm về phía ngược lại vì
nếu không thì về phía có điểm cuối của vector V , tồn tại hai điểm a, b lần lượt
thuộc M1 , M2 thì góc hợp với a và V nhỏ hơn 90◦ . Tương tự, ta thay a vào M2 ,
thay b thay vào M1 thì sẽ có | V + (a − b) |>| V |, mâu thuẫn với tính lớn nhất

28
của | V |.
1 1
Bổ đề 2: Đặt Sk = {(x, y)| | x |= k − , | y |= k − }. Xét l là một đường thẳng
2 2
qua 0 nhưng không qua bất kì điểm nào của Sk . Gọi những điểm thuộc Sk nằm
về một phía nào đó của l là Ak , phía còn lại là Bk . Khi đó, giá trị lớn nhất của
P P
| Vk |=| u∈Ak u − v∈Bk v | đạt được khi l vuông góc với Vk .
Chứng minh: Các điểm của ô nằm vào phía trong hình vuông lớn và dễ dàng
thấy các điểm này lập thành 4 đỉnh của hình vuông với cạnh có độ dài 2k là
1 1 1 1 1 1 1 1
A(k − , k − ), B(−k + , k − ), C(−k + , −k + ), D(k − , −k + )
2 2 2 2 2 2 2 2
Do tính đối xứng, đường thẳng l cắt đoạn thẳng AD và góc nghiêng không âm,
như vậy giao điểm giữa l và AD nằm giữa điểm thứ t và t + 1 của Sk tính từ
trên xuống. Ta có:

− →
− →
− →
− →

Vk = (2k − 2)(2k − 1) j + (2k − t)(−(2k − 1) i + t j ) + t((2k − 1) i + (2k − t) j

− →

= −2(2k − 1)(k − t) i + 2(−(k − t)2 + 3k 2 − 3k + 1) j .

Trong đó i, j là các vector đơn vị. Đặt (k − t)2 = u, 0 ≤ u ≤ k 2 thì


1
| Yk |2 = (2k−1)2 u+u2 −2(3k 2 −3k+1)u+(3k 2 −3k+1)2 = u2 −(2k 2 −2k+1)u+(3k 2 −3k+1)2 .
4
Các biểu thức trên có liên quan đến hàm số bậc hai của u với trục đối xứng là
1 1
u = k 2 − k + nên dễ thấy rằng u = 0, | Vk2 | đạt giá trị lớn nhất hay | Vk đạt
2 4
giá trị lớn nhất. Lúc này, ta cũng có t = k và l vuông góc với Vk .

(+) Trở lại bài toán ban đầu.

Do tính đối xứng, ta chỉ cần xét trường hợp góc nghiêng không âm nhỏ hơn
90◦ , gọi Sk là những tập hợp điểm nằm về hai phía của l.
P P
Đặt M2 ∩ Sk = Ak , M1 ∩ Sk = Bk , Vk = u∈AK u − v∈Bk v thì
1006
X 1006
X 1006
X
| V |=| = Vk |≤ =| Vk |⇒ =| Vk | max .
k=1 k=1 k=1

Xét một đường thẳng l là đường thẳng đi qua 0, song song với các cạnh của
hình vuông thì M2 nằm ở nửa phần trên, M1 nằm ở nửa phần dưới và tất cả
các giá trị | Vk | đều đạt cực đại. Suy ra | Vk | max = 2 · 20063 .
Vậy giá trị lớn nhất cần tìm là 2 · 10063 . Vậy độ dài lớn nhất của tổng các vecto
là 2 · 10063 .

Bài toán 2.1.5. (APMO 2006) Trong cùng một mặt phẳng ta xét n hình tròn
C1 , C2 , . . . , Cn sao cho với mọi i, 1 ≤ i < n, tâm của đường tròn C1 nằm trên chu
vi của đường tròn Ci+1 và tâm của đường tròn Cn nằm trên chu vi của đường

29
tròn C1 . Ta gọi điểm ghi của một sự sắp xếp m hình tròn như thế là các cặp
(i, j) sao cho C1 chứa hoàn toàn trong Cj . Hãy xác định số lớn nhất các điểm
ghi có thể có.

Lời giải. Bài toán cần dự đoán một kết quá để có thể hy vọng định hướng
được lối đi. Bằng cách xét một vài trường hợp riêng của n ta có với n = 2, n = 1
thì rõ ràng không có điểm ghi nào, với n = 3 thì có 1 điểm ghi.
1
Ta có dự đoán kết quả là: (n − 1)(n − 2).
2
Ta gọi một tập gồm n hình tròn thỏa mãn các giả thiết của bài toán là một
n− cấu hình. Cho một n− cấu hình C = (C1 , C2 , . . . , Cn ).
Đặt SC = {(i, j) | C1 chứa hoàn toàn trong Cj }. Như vậy số điểm ghi của n−
cấu hình C là | SC | . Đề chứng minh được dự đoàn của mình, ta sẽ chứng
minh:
1
• Tồn tại một n− cấu hình C sao cho | SC |= (n − 1)(n − 2),
2
1
• | SC |≤ (n − 1)(n − 2), với mọi n− cấu hình C .
2
Cho C1 là hình tròn bất kì. Với mọi u = 1, 2, . . . , n − 1 ta chọn C1 là hình tròn
nằm bên trong Ci−1 sao cho tâm của Ci−1 nằm trên đường trên đường tròn C1 .
Sau cùng gọi Cn là hình tròn có tâm nằm trên đường tròn C1 và chu vi của Cn
chứa tâm Cn−1 .
1
Khi đó ta có: SC = {(i, j) | 1 ≤ i < j ≤ n − 1} và | SC |= (n − 1)(n − 2).
2
Như vậy khẳng định đầu tiên được chứng minh.
Ta chứng minh khẳng định tiếp theo, trước hết cần quan sát và rút ra một số
nhận xét sau:

(1) (i, j) ∈
/ SC ,

(2) (i, 1 + i), (1, n) ∈


/ SC ,

(3) Nếu (i, j); (i, k) ∈ SC thì (i, k) ∈ SC ,

(4) Nếu (i, j) ∈ SC , thì (j, i) ∈


/ SC .

Bây giờ ta sẽ chứng minh rằng nếu một tập G các cặp thứ tự gồm các số
nguyên từ 1 đến n mà G thỏa mãn các điều kiện trên thì khi đó G không có
1
nhiều hơn (n − 1)(n − 2) phần tử. Thực tế ở đây chúng ta đã cụ thể hóa bài
2
toán bằng cách đếm phần tử của một tập hợp khi biết được điều kiện cho
trước.
Giải: Giả sử ngược lại tồn tại một tập G thỏa mãn 4 điều kiện trên mà G có
1
nhiều hơn (n − 1)(n − 2) phần tử. Trong các tập G như thế, ta xét tập G với
2

30
số n bé nhất.
Để ý rằng phải có (i, i + 1) phần tử với I nào đó 1 ≤ i < n hoặc có phần tử
(n, 1) vì nếu không G sẽ chỉ có nhiều nhất số phần từ là:
1 1
Cn2 − n = n(n − 3) < (n − 1)(n − 2).
2 2
Không giảm tổng quát, giả sử (n, 1) ∈ G, lúc đó (1, n − 1) ∈/ G vì nếu không thì
theo (1) suy ra (n, n − 1) ∈ G, mâu thuẫn với (2).
Đặt G0 = {(i, j) ∈ G | 1 ≤ i, j ≤ n − 1}. Vâỵ tập này thỏa mãn điều kiện từ (1)
đến (4).
Ta có | G − G0 |≤ n − 2. Thật vậy, giả sử rằng nếu | G − G0 |> n − 2 ta có
| G − G0 |= n − 1. Suy ra với moị i, 1 ≤ i < n − 1, một trong 2 cặp (i, n) hoặc
(n, i) phải thuộc G.
Để ý rằng (n, 1) ∈ G và (n − 1, 1) ∈ G suy ra (n, n − 2) ∈
/ G, (n − 2, n) ∈ G. Tiếp
tục quá trình này thì ta có: (n, 1) ∈ G. Điều này mâu thuẫn.
Vậy | G − G0 |≤ n − 2, suy ra:
1 1
| G0 ≤ (n − 1)(n − 2) − (n − 2) = (n − 3)(n − 2).
2 2
Tuy nhiên G0 là tập với số n − 1, điều này mâu thuẫn vì ta đang xét tập G với
số n bé nhất. Bài toán được chứng minh hoàn toàn.
1
Vậy số lớn nhất các điểm ghi có thể có là (n − 3)(n − 2).
2
Bài toán 2.1.6. (IMO lần thứ 11-1969) Cho n điểm nằm trong một mặt phẳng
(n > 4), trong đó không có ba điểm nào thẳng hàng. Chứng minh rằng có nhiều
(n − 3)(n − 4)
nhất tứ diện lồi có các đỉnh đỉnh nằm trong số n điểm đó. Cho
2
n điểm trong mặt phẳng, với n > 4, trong số đó không có 3 điểm nào thẳng
(n − 3)(n − 4)
hàng. Chứng minh rằng có ít nhất tứ giác lồi tạo thành có đỉnh
2
nằm trong số n điểm đó.
Lời giải.Trước tiên, ta xét 5 điểm bất kì, không có 3 điểm nào thẳng hàng.Ta
vạch một bao lồi từ 5 điểm đó. Nếu bao lồi này có hơn 3 điểm thì hiển nhiên có
ít nhất 1 tứ giác lồi. Nếu chỉ gồm 3 điểm, chẳng hạn A, B, C thì hai điểm D, E
còn lại sẽ phải nằm trong tam giác ABC . Khi đó, có hai đỉnh của tam giác
ABC nằm cùng phía đối với đường thẳng DE , và cùng với D, E hai đỉnh đó
tạo thành một tứ giác lồi. Như vậy, mệnh đề cần chứng minh đúng với n = 5
(5 − 3)(5 − 4)
(để ý = 1).
2
Xét n điểm với n > 5. Vì không có 3 điểm nào thẳng hàng nên số tất cả các
n(n − 1)(n − 2)(n − 3)(n − 4)
cách chọn n điểm như trên là: Cn5 = = 1. Mỗi cách
2
chọn này cho ta ít nhất 1 tứ giác lồi. Tuy nhiên, bất kì tứ giác lồi nào trong số

31
đó cũng có thể được lập từ (n − 4) tập hợp khác nhau gồm 5 điểm nói trên. Do
1 n(n − 1)(n − 2)(n − 3)
vậy có ít nhất · Cn5 = số tất cả các tứ giác lồi được
n−4 120
thành lập từ n điểm đã cho. Để kết thúc chứng minh, ta sẽ chứng tỏ rằng:
n(n − 1)(n − 2)(n − 3) (n − 3)(n − 4)
≥ ∀x ≥ 5.
120 2
Điều phải chứng minh này tương đương với n(n − 1)(n − 2) ≥ 60(n − 4).
Dễ dàng thấy rằng các số n = 5 và n = 6 là nghiệm của phương trình

n(n − 1)(n − 20 − 60(n − 4) = 0.

Do đó ta có thê phân tích thành nhân tử như sau

n(n − 1)(n − 20 − 60(n − 4) = (n − 5)(n − 6)(n + 8).

Với mọi n nguyên dương và n > 5, lập bảng xét dấu. Dễ thấy dấu của biểu
thức (n − 5)(n − 6)(n + 8) cũng là dấu của (n − 5)(n − 6), từ đó ta suy ra
• (n − 5)(n − 6) = 0 khi n = 5, 6;

• (n − 5)(n − 6) > 0 khi n > 6 suy ra điều phải chứng minh.

Bài toán 2.1.7. (IMO lần thứ 16-1974) Một bàn cờ 8 × 8 được chia thành p
hình chữ nhật rời nhau (theo đường lưới giữa các ô vuông) sao cho mỗi một
hình chữ nhật sẽ có số ô trắng bằng với số ô đen, và các hình có số ô vuông
khác nhau. Tìm giá trị lớn nhất có thể của p và tất cả các bộ có thể của kích
thước các hình chữ nhật.
Lời giải. Điều kiện mỗi hình chữ nhật có sô các ô trắng bằng số các ô đen có
nghĩa là mỗi hình chữ nhật có một số chẵn các ô vuông. Ta có:

2 + 4 + 6 + 8 + 10 + 12 + 14 + 16 = 72 > 64,

do đó phải có p < 8. Có 5 khả năng phân số 64 thành tổng của 7 số không bằng
nhau: Khả năng đầu tiên bị loại bỏ vì một hình chữ nhật có 22 ô vuông thì

phải có một cạnh dài hơn 8 (đơn vị ô). Các khả năng còn lại chấp nhận được,
cụ thể, sự phân chia như sau:

32
Bài toán 2.1.8. (IMO lần thứ 38-1997) Trong mặt phẳng có các điểm với
toạ độ nguyên là đỉnh của các hình vuông đơn vị. Các hình vuông này được
tô màu xen kẽ trắng đen giống như trên bàn cờ. Với bất kì cặp số nguyên
dương (m, n) xét tam giác vuông có các đỉnh có toạ độ nguyên và hai cạnh bên
có độ dài là m, n nằm dọc theo các cạnh của các hình vuông. Gọi S1 là tổng
diện tích phần đen của tam giác, và S2 là tổng diện tích phần trắng. Và đặt
f (m, n) =| S1 − S2 |.

(a) Tính f (m, n) với mọi số nguyên dương m, n mà hoặc là cả 2 đều chẵn hoặc
là cả 2 đều lẻ.
max(m, n)
(b) Chứng minh rằng f (m, n) ≤ với mọi m, n.
2
(c) Chỉ ra không tồn tại một hằng số C sao cho f (m, n) < C với mọi m, n.

Lời giải.

(a) Nếu cả m lẫn n cùng chẵn thì f (m, n) = 0. Thật vậy, gọi M là trung điểm
cạnh huyền của tam giác. M là một điểm nằm trên mắc lưới (của hình
bàn cờ). Nếu ta xoay tam giác 180◦ thì màu sắc vẫn y nguyên. Do đó, các
số S1 , và S2 của tam giác có giá trị tương ứng bằng một nửa số ô được tô
màu đen, trắng của hình chữ nhật tạo thành. Mà số các ô đen, trắng của
hình chữ nhật băng nhau, suy ra f (m, n) = 0.
1
Nếu m, n đều lẻ thì f (m, n) = . Thật vậy, trung điểm M của cạnh huyền
2
tam giác bây giờ là tâm của một hình vuông, ta thấy rằng màu của hai
nửa tam giác giống nhau.
Lúc này, S1 và S2 của tam giác khác hình chữ nhật như đã nói trên 1 đơn
1
vị nên thì f (m, n) = . Nếu m, n cùng chẵn hoặc cùng lẻ thì kết quả có
2

33
được lập tức từ câu (a). Giả sử n chẵn và m lẻ. Khi đó, ta kéo dài cạnh có
độ dài m thêm 1 đơn vị nữa thì ta được một tam giác mới chứa tam giác
cũ. Tam giác mới này có cả hai cạnh cùng chẵn nên có S1 = S2 . Diện tích
được thêm vào là điện tích một tam giác có một cạnh góc vuông bằng 1,
n
chiều cao n nên diện tích thêm vào bằng . Trường hợp xấu nhất, diện tích
2
1
này được phủ chỉ bởi một màu thì ta có f (m, n) = nhưng n < max(m, n)
2
nên ta suy ra điều phải chứng minh.

(c) Bằng trực giác, ta thấy rằng nếu cạnh huyền chạy dọc theo đường chéo
của một dãy các hình vuông màu đen thì khi chúng ta kéo dài đường chéo
ấy về một phía, phần diện tích chiếm chỗ thêm vào sẽ có màu đen nhiều
hơn. Ta sẽ chứng minh điều này bằng lí luận.
Trong trường hợp này ta có n = m. Khi mở rộng cạnh huyền về một phía
như đã nói, phần màu trắng thêm vào là một dãy các tam giác vuông mà
mỗi tam giác này đồng dạng với một tam giác mới có các cạnh góc vuông
n
là (n + 1) và n. Tam giác lớn nhất có cạnh là 1 và , tam giác lớn
n+1
n−1 n−1
tiếp theo có cạnh là , tiếp theo nữa là tam giác có các cạnh
n n+1
n−2 n−2
, , . . . , và sau cùng, tam giác nhỏ nhât có các cạnh góc vuông
n n+1
1 1
là , .
n n+1
Suy ra điện tích phần màu trắng thêm vào là
1 n (n − 1)2 (n − 2)2 1 1 2n + 1
( + + +· · ·+ )= (n2 +· · ·+12 ) = .
2 n + 1 n(n + 1) n(n + 1) n(n + 1) 2n(n + 1) 12
n 2n + 1 n 1
Từ đó, diện tích phần màu đen được thêm vào là − = − nó
2 12 3 12
nhiều hơn diện tích phần màu trắng thêm vào là
n 1 2n + 1 n 1 n−1
− − = − = .
3 12 12 6 6 6
Đến đây, ta thấy rằng nếu n là số chẵn thì f (n, n) = 0 đối với tam giác
n−1
ban đầu, do vậy tam giác mới sẽ có f (n + 1, n) = không thể bị chặn
6
theo n được, từ đó ta suy ra điều phải chứng minh.

Bài toán 2.1.9. (Vietnam TST 1993) Gọi hình chữ nhật 2 × 3 (hoặc 3 × 2) bị
cắt bỏ một ô vuông 1 × 1 ở góc được gọi là hình chữ nhật khuyết đơn (Hình
1). Hình chữ nhật 2 × 3 (hoặc 3 × 2) bị cắt bỏ hai hình vuông 1 × 1 nẳm ở hai
góc đối diện là hình chữ nhật khuyết kép (Hình 2). Người ta ghép một ô vuông
2 × 2, một số hình chữ nhật đơn và một số hình chữ nhật khuyết kép sao cho
không có hai hình nào chờm lên nhau để tạo thành một hình chữ nhật kích

34
thước 1993 × 2000. Gọi s là tổng hình vuông 2 × 2 và hình chữ nhật khuyết kép
trong mỗi cách ghép trên. Tìm giá trị lớn nhất của s.

Lời giải.Gọi y là số hình chữ nhật khuyết đơn. Ta có đẳng thức về diện tích
4s + 5y = 2000 × 1993 (1)
Điền số 0 vào các ô (2k, 2t) với 1 ≤ k ≤ 996, 1 ≤ t ≤ 1000, ta thấy có 1000, 996

số 0 được điền. Dựa vào Hình trên ta thấy: Hình vuông 2 × 2 và hình chữ nhật
khuyết kép chứa đúng một số 0. Hình chữ nhật khuyết đơn chứa x số 0 với
x = 1 hoặc x = 2. Suy ra

100, 96 = 1s + xy ≥ s + y (2)
Từ (1) và (2) ta suy ra được
2000 · 1993 = 5(s + y) − s ≤ 5 · 1000 · 996 − s.

Suy ra s ≤ 99400. Ta xét cách ghép sau thỏa mãn s = 994000.


Vậy giá trị lớn nhất của tổng số hình vuông 2 và hình chữ nhật khuyết kép
trong mỗi cách ghép nó là 994000.

35
2.2 Bài toán về tìm giá trị nhỏ nhất
Bài toán 2.2.1. (Romania Masters in Mathematics 2015) Cho một số nguyên
dương n, xác định số thực lớn nhất µ, thỏa mãn điều kiện sau: Với bất kì tập
C gồm 4n điểm bên trong một hình vuông U luôn tồn tại một hình chữ nhật
T nằm trong U sao cho

• Các cạnh của T song song với các cạnh của U .

• Phần trong của T chứa đúng một điểm của C .

• Diện tích của T nhỏ nhất là µ.


1
Lời giải. Ta dự đoán đáp số là µ = . Điều này xuất phát trong trường
2n + 2
k
hợp mà C là tập các điểm (1/2 ± ε, ± ε) với 1 ≤ k ≤ n và ε > 0 rất nhỏ.
n+1
Gọi x1 < · · · < xk là các tọa độ theo trục Ox của các điểm thuộc C , và gọi ai
là số điểm mà tọa độ trục Ox của nó là xi . Ngoài ra đặt x0 = 0; xk+1 = 1.
Giả sử rằng với mọi hình T thỏa mãn hai điều kiện đầu thì diện tích của nó
1
nhỏ hơn µ = .
2n + 2
a
Với mọi 1 ≤ i ≤ k , miền xi−1 ≥ x ≤ xi+1 , 0 ≤ y ≤ 1 có thể được bao phủ [ i ] + 1
2
hình chữ nhật thỏa mãn hai điều kiện đầu. Nếu y1 < · · · < yai là các tọa độ
theo trục 0y và cho các hình chữ nhật phủ các khoảng y ∈ (0, y2 ) · (y2 y4 ), . . . và
x ∈ (0; x2 ) . . . (x2 x4 ). Khi đó ta dễ dàng chứng minh được là mỗi hình chữ nhật
chứa đúng một điểm. Do diện tích của mỗi hình chữ nhật nhỏ hơn µ, ta cộng
các diện tích lại và ta được
ai
xi+1 − xi−1 < ([ ] + 1)µ.
2
Nếu k = 2t + 1, ta được

1 = (x2 −0)+(x4 −x2 )+· · ·+(x2t −x2t−2 )+(1−x2t ) < f (ai )+f (a3 )+· · ·+f (a2t+1 )µ.

1 < (x2 −0)+(x3 −x1 )+(x5 −x3 )+· · ·+(x2t −x2t−2 )(1−x2t ) < f (ai )+f (a3 )+· · ·+f (a2t+1 )µ.

Cộng từng vế ta được:

2 < (2f (a1 ) + f (a2 ) + f (a3 ) + · · · + f (a2k−1 ) + k)µ (1)

Nếu k = 2t, làm tương tự như trên ta được

1 < (f (a1 ) + f (a2 ) + f (a4 ) + · · · + f (a2t−2 ) + f (a2t ))µ.

1 < (f (ai ) + f (a3 ) + · · · + f (a2t+1 ))µ.

36
Cộng từng vế lại ta được bất đẳng thức (1) như trường hợp trước (k = 2t + 1).
x
Do 2f (x) ≤ 2( + 1) = x + 2 và f (x) ≤ x với mọi số nguyên x ≥ 1, kết hợp với
2
(1) ta có

2 < (a1 + 2 + a3 + · · · + ak−1 + ak+2 )µ = (4n + 4)µ = 2.


1
Rõ ràng mâu thuẫn. Vậy giả thiết trên của ta là sai và ta thu được µ =
2n + 2
là số cần tìm.

Bài toán 2.2.2. (VMO-2007) Cho 1 đa giác đều 2007 đỉnh. Tìm số nguyên
dương k nhỏ nhất thỏa mãn tính chất: trong mỗi cách chọn k đỉnh của đa giác
luôn tồn tại 4 đỉnh tạo thành 1 tứ giác lồi mà 3 trong số 4 cạnh của nó là 3
cạnh của đa giác đã cho.

Lời giải. Gọi các đỉnh của đa giác đều là: A1 , A2 , . . . , A2007 . Chú ý là tứ giác
(tạo từ 4 trong số các đỉnh của đa giác) có 3 cạnh là 3 cạnh của đa giác khi và
chỉ khi 4 đỉnh của tứ giác đó là 4 đỉnh liên tiếp của đa giác.
Gọi A là tập các đỉnh: {A1 , A2 , A3 , A5 , . . . , A2006 } bỏ đi các đỉnh A4i với i =
1, . . . , 501 và A2007 .
Ký hiệu | A | là số phần tử của tập hợp A. Hiển nhiên | A |= 1505 và trong
A không chứa 4 đỉnh liên tiếp nào của đa giác. Dễ thấy, mọi tập con của
A đều không chứa 4 đỉnh liên tiếp của đa giác. Vậy k ≥ 1506. Ta sẽ chứng
minh cách chọn 1506 đỉnh tùy ý của đa giác thì sẽ tồn tại 4 đỉnh liên tiếp
của đa giác trong 1506 đỉnh đó. Thật vậy, giả sử T là 1 tập hợp gồm 1506
đỉnh tùy ý của đa giác. Phân hoạch tập các đỉnh của đa giác thành tập hợp
B1 = {A1 , A2 , A3 , A4 }; . . . ; B2 = {A1 , A2 , A3 , A4 }B502 {A2005 , A2006 , A2007 }. Giả sử
T không chứa 4 đỉnh liên tiếp của đa giác. Lúc đó với mỗi i = 1, . . . , 501.
Tập Bi không thuộc T , tức là mỗi tập Bi đó sẽ có ít nhất 1 đỉnh không thuộc
T . Khi đó: | A |≤ 3 · 502 = 1506.
Do | A |= 1506 nên B502 ⊂ T và mỗi tập Bi với i = 1, . . . , 501 có đúng 3 phần tử
thuộc T .
Ta có A2005 , A2006 , A2007 ∈ T ⇒ A1 ∈ / T.
Suy ra A2 , A3 , A4 ∈ T ⇒ A5 ∈ / T.

⇒ A6 , A7 , A8 ∈ T · · · ⇒ A2002 , A2003 , A2004 ∈ T.

Khi đó 4 đỉnh liên tiếp A2002 , A2003 , A2004 , A2007 ∈ T . Mâu thuẫn. Vậy suy ra
k = 1506.
Vậy số nguyên dương k nhỏ nhất thỏa mãn tính chất cần tìm là 1506.
Nhận xét: Có thể giải ngắn gọn bằng cách xét 2007 − 1506 = 501 điểm còn lại

37
chia đường tròn ngoại tiếp đa giác đều đã cho không quá 501 cung, và phải có
1506
1 cung trong chúng chứa không ít hơn > 3 đỉnh liên tiếp.
501
Bài toán 2.2.3. (Dự tuyển VMO-2012) Cho M là tập gồm n điểm trong mặt
phẳng thỏa mãn:
• Tồn tại 7 điểm thuộc M là 7 đỉnh của một thập giác lồi.

• Với bất kì 5 điểm nào thuộc M mà là 5 đỉnh của một ngũ giác lồi luôn
có một điểm nữa của M nằm hoàn toàn trong ngũ giác đó (phần trong
ngũ giác được hiểu là phần mặt phẳng bên trong ngũ giác, không kể các
cạnh).
Hãy tìm giá trị nhỏ nhất của n?
Lời giải. Ta chứng minh n ≥ 11. Gọi thất giác có 7 đỉnh thuộc M là A1 A2 A3 . . . A7 .
Từ giả thiết (2) tồn tại P1 thuộc M và P1 nằm trong ngũ giác A1 A2 A3 A4 A5 .
Nối P1 A1 và P1 A5 ta suy ra tồn tại P2 ∈ M , P2 nằm trong ngũ giác A1 P1 A5 A6 A7
và P1 6= P2 . Do đó có ít nhất 5 điểm thuộc {A1 , A2 , A3 , . . . , A7 } không nằm trên
đường thẳng P1 P2 . Theo nguyên lý Dirichle có ít nhất 3 điểm nằm trên cùng
nửa mặt phẳng bờ P1 P2 và 3 điểm này cùng với P1 P2 tạo thành ngũ giác, ngũ
giác này chứa điểm P3 ∈ M.
Bây giờ ta có 3 đường thẳng P1 P2 ; P2 P3 ; P3 P1 tạo thành tam giác P1 P2 P3 . Ký
hiệu π1 là nửa mặt phẳng bờ P1 P2 không chứa tam giác P1 P2 P3 . Ký hiệu tương
tự cho π2 và π3 . Tổng diện tích π1 + π2 + π3 phủ toàn bộ mặt phẳng trừ tam giác
P1 P2 P3 . Theo nguyên lý Dirichle tồn tại 1 trong 3 miền chứa ít nhất 3 điểm
{A1 , A2 , A3 , . . . , A7 }, không giảm tính tổng quát ta giả sử π1 chứa A1 A2 A3 . Khi
đó tồn tại P4 ∈ M nằm trong ngũ giác tạo bới A1 , A2 , A3 , P1 , P2 . Vậy n ≥ 11.
Ta chỉ ra một ví dụ chứng tỏ n = 11 là thỏa mãn. Ta chọn M gồm 7 điểm là
đỉnh một thật giác lồi và 4 điểm nằm trong, 11 điểm này đều có tọa độ nguyên
thuộc M nằm ở trong nó. Khi đó trong số ngũ giác như vậy có 1 ngũ giác có
diện tích nhỏ nhất là ABCDE .
Vì có 4 trường hợp về tọa độ điểm nguyên là (lẻ, chẵn), (chẵn, lẻ), (lẻ, lẻ),
(chẵn, chẵn) nên phải tồn tại 2 đỉnh trong số 5 đỉnh A, B, C, D, E cùng tính
chẵn lẻ về tọa độ. Suy ra trung điểm của đoạn nối 2 đỉnh đó, gọi là P , cũng là
1 điểm nguyên thuộc M và P không nằm trong ABCDE . Suy ra P nằm trên
cạnh của ngũ giác. Giả sử P ∈ AB thì P phải là trung điểm của AB , suy ra
P BCDE là ngũ giác lồi có diện tích nhỏ hơn thật sự diện tích ABCDE (vô lý).
Vậy giá trị nhỏ nhất của n là 11.
Bài toán 2.2.4. (VNTST – 2006) Trong không gian cho 2006 điểm mà trong
đó không có 4 điểm nào đồng phẳng. Người ta nối tất cả các điểm đó lại bởi

38
các đoạn thẳng. Số tự nhiên m gọi là số tốt nếu ta có thể gán cho mỗi đoạn
thẳng trong các đoạn thẳng đã nói bởi một số tự nhiên không vượt quá m sao
cho mỗi tam giác tạo bởi ba điểm bất kì trong số các điểm đó đều có hai cạnh
được gán bởi hai số bằng nhau và cạnh còn lại gán bởi số lớn hơn hai số đó.
Tìm số tốt có giá trị nhỏ nhất.
Lời giải.Cũng như nhiều bài toán khác, ở bài này, ta thấy số 2006 không có ý
nghĩa lớn lắm và thử tùy ý tổng quát trong trường hợp n = 2 tùy ý. Tư tưởng
“chia nhị phân” là một trong các đại diện quan trọng của ứng dụng chiến lược
chia để trị. Thay vì xử lý bài toán lớn, ta chia nó ra và giải quyết từng phần
nhỏ.
Ở bài này, ta sẽ chỉ ra một tình huống mà khi không phân tích thấu đáo, dựa
theo các xây dựng cục nook và thử với vài số nhỏ, ta sẽ dễ dàng rơi vào một
ngộ nhận nào đó dẫn đến kết quả sai. Ta cũng tiến hành tương tự như các ví
dụ đã nêu. Xét các tình huống với số nhỏ:
• Với n = 2 thì cần m = 0 là được,

• Với n = 3 thì cần đến 2 số để đánh nên chọn m = 1,

• Với n = 4 thì cũng chỉ cần 2 số nên m = 1,

• Với n = 5 thì ta cần 3 số và m = 2.

Đến đây dễ thấy rằng luôn tồn tại một cạnh nào đó được đánh số 0. Giả sử
cạnh nối A và B . Ta chia n − 2 đỉnh còn lại thành 2 phần rõ ràng mỗi đỉnh đó
đều phải nối với A hoặc B bởi cạnh đánh số 0. Ta lại chia chúng thành 2 tập
hợp: X chứa các đỉnh nối với A bởi cạnh đánh số 0, Y chứa các đỉnh nối với B
bởi các cạnh đánh số 0. Khi đó ta có thể cho:
• Từ mỗi đỉnh tập X sang mỗi đỉnh tập Y , cạnh nối với nhau đánh số 0,

• Từ đỉnh A sang tập Y và từ đỉnh B sang tập X , cạnh nối với nhau đánh
số 1.
Còn lại trong X và Y , ta cần sử dụng max{(f | X |, f (| Y |)}. Tuy nhiên, do
cần chọn nhỏ nhất nên:

f (n) = 2 + min{min{(f | X |, f (| Y |))}}.

Hơn nữa, f (n) là hàm đơn điệu và | X |, | Y |= n − 2 nên


n−1
min{max{(f | X |, f (| Y |))}} = f ([ ]).
2
Lập luận có vẻ phù hợp nhưng đáng tiếc là kết luận này lại sai và với phần ví
dụ khi xây dựng trong trường hợp n = 7, n = 8 ta dễ dàng phát hiện ra vấnn

39
đề nằm ở chỗ cách xây dựng mô hình. Trên thực tế, ta có thể chia ra ngay từ
đầu chứ không cần phải xét thêm điểm A, B nên công thức đúng là
n−1
f (n) = 2 + f ([ ]).
2
n−1
Vậy giá trị nhỏ nhất là 2 + f ([ ]).
2
Bài toán 2.2.5. [3] Cho n điểm trên mặt phẳng, với n > 4 và không có ba
(n − 3)(n − 4)
điểm nào thẳng hàng. Chứng minh có ít nhất tứ giác lồi có các
2
đỉnh nằm trong n điểm đã cho.
Lời giải. Trước hết xét 5 điểm bất kì, không có 3 điểm nào thẳng hàng, nếu
bao lồi của hình này có nhiều hơn ba điểm thì luôn tồn tại ít nhất một tứ giác
lồi từ 5 điểm A, B, C, D, E . Chẳng hạn tứ giác ABCD. Nếu hình bao chỉ có ba
điểm A, B, C vậy hai điểm D, E nằm trong tam giác ABC . Suy ra khi đó hai
đỉnh của tam giác ABC nằm về một phía với đường thẳng DE .
Tứ giác lồi được tạo thành từ năm điểm là BDEC , mệnh đề chứng minh đúng
với n = 5.
(n − 3)(n − 4)
Với n = 5 ⇒ = 1.
2
Xét n > 5, theo giả thiết không có ba điểm nào thẳng hàng.
Theo công thức tính tổ hợp ta có tất cả các cạnh chọn trong n điểm là:
n(n − 1)(n − 2)(n − 3)(n − 4)
Cn5 = .
120
Mỗi cách chọn này cho ta ít nhất một tứ giác lồi, trong bất kì tứ giác lồi nào
trong số đó cũng đều lập ra từ (n − 4) tập hợp khác nhau từ 5 điểm nói trên.
Vậy có ít nhất
1 n(n − 1)(n − 2)(n − 3)(n − 4)
Cn5 = .
n−4 120
Số tứ giác lồi được hình thành từ n điểm đã cho. Bài toán đua về chứng minh
bất đẳng thức
n(n − 1)(n − 2)(n − 3)(n − 4) (n − 3)(n − 4)

120 2
với mọi n ≥ 5.
• n(n − 1)(n − 2)(n − 3) ≥ 60(m − 4),

• (n − 5)(n − 6) ≥ 0,

• n > 6.
(n − 3)(n − 4)
Từ đó kết luận: Luôn tồn tại ít nhất tứ giác lồi có các đỉnh nằm
2
trong n điểm đã cho.

40
Bài toán 2.2.6. [3] Cho n điểm bên trong hình vuông ABCD có cạnh bằng
1. Chứng minh rằng tồn tại tam giác có đỉnh tại các điểm đã cho hoặc là đỉnh
1
hình vuông sao cho diện tích tam giác không vượt quá .
2(n + 1)
Lời giải. Gọi n điểm đã cho nằm trong hình vuông A1 , A2 , A3 , . . . , An suy ra
được 4 tam giác.
Nếu A2 nằm một trong bốn tam giác đó, chẳng hạn tam giác A1 CD, nối A2
với các đỉnh A1 , C, D. Số tam giác tạo thành tăng thêm 2.
Nếu A2 nằm trên một tam giác A1 CD, chảng hạn nằm trên A1 C , nối A2 với
các đỉnh B và D, như vậy số tam giác được tạo thành cũng tăng thêm 2.
Vậy mọi trường hợp tam giác đều tăng thêm 2.
Với các điểm A1 , A2 , A3 , . . . , An ta làm tương tự, số tam giác mỗi lần được tạo
tành thêm 2, sau n bước ta thu được tổng các tam giác tạo thành: 4+(n−1)2 =
2n + 2 tam giác. Các tam giác này được tạo bởi các đỉnh đã cho hoặc các đỉnh
A, B, C, D.
Tổng số diện tích 2n + 2 tam giác được tạo thành bằng tổng diện tích hình
vuông ABCD.
Gọi diện tích của 2n + 2 tam giác là S1 , S2 , . . . , Si , . . . , S2n+2 . Suy ra
S1 + S2 + · · · + Si + · · · + S2n+2 = 1.

Tồn tại
1
Sk = min{S1 , S2 , . . . , Si , . . . , S2n+2 } ⇒ S ≤ .
2(n + 1)
Bài toán 2.2.7. (IMO lần thứ 31-1990) Lấy n ≤ 3 và xét một tập E gồm
2n − 1 điểm khác nhau trên một đường tròn. Giả sử rằng chính xác trong đó có
k điểm được tô màu đen. Cách tô màu được gọi là tốt nếu tồn tại ít nhất môt
cặp điểm đen sao cho bên trong một trong hai cung giữa chúng chứa chính xác
n điểm thuộc E . Tìm giá trị nhỏ nhất của k sao cho với mọi cách tô màu k
điểm của E như thể đều tốt.
Lời giải. Theo chiều kim đồng hồ trên đường tròn, ta kí hiệu 2n − 1 điểm đã
cho lần lượt bởi 0, 1, 2, . . . , 2n − 2.
Đặt K = {0, 1, 2, . . . , 2n − 2}. Với m nguyên ta đồng nhất m với I thuộc K khi
m = i (mod 2n − 1). Ta định nghĩa quan hệ i ∗ j đối với hai phân tử i và j
(thuộc K ) nếu ta có | i − j |= n + 1 hoặc | i − j |= n − 2.
Rõ ràng ta có i ∗ j khi và chỉ khi một trong hai phần trong của cung tạo thành
chứa đúng n điểm của E . Nếu k là một số thỏa mãn tính chất của đầu bài thì
ta nói đơn giản rằng k là tốt. Từ đó, k là tốt khi và chỉ khi với mọi tập con H
của K gồm k phần tử, tồn tại hai phần tử của H có quan hệ ∗ với nhau. Ta sẽ
chứng minh:

41
(a) Số n là tốt,
(b) Nếu 2n − 1 không chia hết cho 3 thì n − 1 không phải tốt,
(c) Nếu 2n − 1 chia hết cho 3 thì n − 1 là tốt.
Chứng minh (a): Xét tập con H của K gồm n phần tử. Giả sử trong H không
có cặp phần tử nào có quan hệ ∗ với nhau. Với j thuộc H , ta kí hiệu K(j) là
tập hợp tất cả các phân tử (của K ) có quan hệ ∗ với j . Thế thì K(j) và H
không giao nhau với mọi j thuộc H . Vì mỗi phần tử có mặt nhiều nhất ở trong
hai K(j) nên hợp các K(j) (với j chạy khắp H ) có ít nhất n phần tử, điều này
mâu thuẫn với việc H có n phần tử.
Chứng minh (b): Để chứng minh khẳng định (b), ta chỉ cần xây dựng một tập
con L của K sao cho L gồm n − 1 phần tử và không có cặp nào có quan kệ ∗
với nhau.
Xét các tập L = {3k | k = 0, 1, . . . , n − 2}.
M = {3k + n − 2 | k = 0, 1, . . . n − 1}
= {3(n − 1 − k) + n − 2 | k = 0, 1, . . . , n − 1}
= {4n − 5 − 3k | k = 0, 1, . . . , n − 1}
= {3(−1 − k) | k = 0, 1, . . . , n − 1}.

Vì 2n − 1 không chia hết cho 3 nên ta suy ra L giao M = {3k | k = −n, −n +


l, . . . , n − 2} = K.
Từ đó ta có L gồm n − 1 phần tử, M gồm n phần tử, L và M có giao bằng
rỗng. Đề ý rằng với j thuộc L, J = 3k trong đó 0 ≤ k ≤ n − 2 và
K(j) = {3k + n − 2, 3k + n + 1} = {3k + n − 2, 3(k + 1) + n − 2} ⊂ M.

Ta kết luận rằng K(j) và L không giao nhau. Điều đó có nghĩa là không có cặp
phần tử nào có quan hệ ∗ với nhau trong L.
Bây giờ xét trường hợp 2n − 1 chia hết cho 3. Trong trường hợp này n ≡ 2 (mod
3). Đặt n = 3m − 1 (với m ≥ 2). Thế thì 2n − 1 = 3(2m − 1), n − 2 = 3(m − 1) và
n + 1 = 3m. Nếu i và j (thuộc K ) có quan hệ ∗ với nhau thì i và j đồng dư với
nhau theo modulo 3. Ta viết:
K = {3k + r | k = 0, 1, . . . , 2m − 2} (r = 0, 1, 2).

Như thế, K được phân thành ba tập con không giao nhau như K0 , K, K2 . Với
mỗi i thuộc K , thì từ I thuộc K , ta suy ra hai phần tử có quan hệ ∗ với i cũng
thuộc K . Ta chứng minh rằng 3m − 2 là tốt
Thật vậy, nếu A là tập con của K và gồm 3m − 2 phần tử thì theo nguyên
tắc Dirichlet, một tập con dạng A ∩ K1 có ít nhất m phần tử. Tương tự như

42
chứng minh ở phần (a) ta đi đến kết luận rằng tồn tại một cặp phần tử có
quan hệ ∗ với nhau trong A ∩ Kr CA.
Bây giờ ta chứng minh rằng 3(m − 1) không tốt. Muốn thế, ta chỉ cần xây dựng
một tập con gồm (m − 1) phần tử của K , mà không có cặp phần tử nào có
quan hệ ∗ với nhau. Đặt M = {3k + r | k = 0, 1, 2, . . . , m − 2}, với r = 0, 1, 2.
Đề ý rằng i và j có quan hệ ∗ với nhau khi và chỉ khi | i − j |= 3(m − 1) hoặc
| i − j |= 3m. Ta thấy ngay không có cặp phần tử nào có quan hệ ∗ với nhau
trong mỗi tập M0 , M1 , M2 . Tóm lại, từ những điều trên, ta suy ra giá trị k nhỏ
nhất (k min) sao cho mọi cách tô màu k điểm của E như đã nói ở đề bài đều
tốt là:

n khi 2n − 1 6= 3m, m ∈ N
kmin =
n − 1 khi 2n − 1 = 3m, m ∈ N.

 
0 (mod3)
n khi n=

kmin = 1 (mod3)
n−1 khi n ≡ 2 (mod3)

2.3 Bài toán liên quan đến cực trị hình học tổ hợp
Bài toán 2.3.1. (IMO lần thứ 23 − 1982) Xét hình vuông S có cạnh dài 100
(đơn vị). Gọi L là một đường gấp khúc nằm trong S không tự cắt chính nó, L
bao gồm các đoạn thẳng A0 A1 , A1 A2 , . . . , An−1 An , với A0 không trùng với An .
Giả sử với mọi điểm P nằm trên biên (tức là các cạnh) của S đều có một điểm
1
trên L sao cho khoáng cách từ điểm này đến P không lớn hơn . Chứng minh
2
rằng tồn tại 2 điểm X và Y trên L sao cho khoảng cách giữa X và Y không
lớn hơn 1, và độ dài phần đường gấp khúc của L nằm giữa X và Y không bé
hơn 198.

Lời giải. Như lệ thường, ta kí hiệu d là khoảng cách Euclide giữa hai điểm.
Khi đi đọc theo đường L, ta chọn một điểm P có khoảng cách đến đỉnh V1 của
1
hình vuông không quá . Ta tiếp tục đi xa hơn cho đến khi tiếp cận với một
2
trong hai đỉnh kề của V1 , gọi đỉnh này là V2 , tương ứng ta cũng có P2 và rồi ta
cũng có V3 , V4 , tương ứng có các điểm P3 , P4 .
Kí hiệu [L < P ] là phần đường L tính từ P về trước và [L > P ] là phần còn lại.
Xét cạnh V1 V2 . Đặt
1
A = {x ∈ V1 V4 : d(x, [L < P2 ]) ≤ },
2
1
B = {x ∈ V1 V4 : d(x, [L < P2 ]) ≤ }.
2

43
S
Chú ý rằng từ giả thiết ta có A B = V1 V4 . Hơn nữa, ta cũng có A ∩ B 6= 0.
1
Chọn z ∈ A ∩ B . Lấy X ∈ [L < P2 ] sao cho d(X, z) ≤ . Cũng như vậy, ta chọn
2
điểm Y ∈ [L < P2 ] Khi đó(X, Y ) ≤ 1 và phần đường gấp khúc giữa X và Y .

Bài toán 2.3.2. (IMO lần thứ 7 − 1965) Cho n điểm trong mặt phẳng (n > 2).
Chứng minh rằng: có nhiều nhất n cặp điểm là có khoảng cách lớn nhất (giữa
hai điểm bất kỳ). Trước hết, ta để ý rằng nếu có hai đoạn thẳng có độ dài
bằng nhau d mà không cắt nhau thì ta có thể tìm hai đầu mút trong 4 đầu
mút của hai đoạn thẳng đó để tạo thành một đoạn thẳng có độ dài lớn hơn d.

Lời giải. Thật vậy, giả sử hai đoạn thắng đó là P Q và RS , với P Q = RS = d.


Nếu điều ngược lại với khẳng định trên xảy ra, ta có góc P[ OR > 90◦ (vì nếu
không thì P R > P Q = d).
Tương tự, ta chứng minh được tất cả các góc của tứ giác đều bé hơn 90◦ ,
điều này mâu thuẫn.
Ta dùng kết quả bài 2.3.1 để chứng minh bài toán. Giả sử có hơn n cặp
điểm có khoảng cách cực đại là d. Như thế, dễ thấy rằng tại một đỉnh A nào
đó phải có 3 đoạn AB, AC, AD có độ dài là d (cực đại). Giả sử AC nằm giữa
AB và AD. Khi đó C không thể nối được với một điểm nào nữa, mâu thuẫn
này cho ta điều phải chứng minh (thậy vậy, giả sử C nối với X , theo Mệnh đề
trên suy ra CX phải cắt AB lẫn AD, mâu thuẫn).

Bài toán 2.3.3. (VMO-2011) Cho ngũ giác lồi ABCDE có các cạnh và hai

đường chéo AC , AD có độ dài không vượt quá 3. Trong ngũ giác này lấy 2011
điểm bất kì. Chứng minh rằng tồn tại một đường tròn đơn vị có tâm nằm trên
cạnh ngũ giác ABCDE và chứa ít nhất 403 điểm trong số 2011 điểm đã cho.

Lời giải. Trước hết ta chứng minh Bổ đề sau: Cho điểm 1 nằm trong tam giác

XY Z có độ dài các cạnh không vượt quá 3. Khi đó ta có:

min{IX, IY, IZ} ≤ 1.

Thật vậy, vì X
[ IY + Yd
IZ + Z
[ IX = 360◦ nên trong ba góc X
[ IY , Yd
IZ, Z
[ IX phải
có một góc không nhỏ hơn 120◦ .
Giả sử XIY ≤ 120◦ thì áp dụng định lý Cosin trong tam giác IXY ta có:

3 ≥ XY 2 = IX 2 +IY 2 −2·IX·IY ·cos X


[ IY ≥ IX 2 +IY 2 +IX·IY ≥ 3 min{IX 2 ; IY 2 }.

Từ đây ta suy ra: min{IX; IY } ≤ 1. Bổ đề được chứng minh.


Quay lại bài toán ta có: Theo giả thiết thì các tam giác ABC, ACD, ADE

đều có ba cạnh không vượt quá 3, mà mỗi điểm trong 2011 điểm được đặt
trong ngũ giác ABCDE đều thuộc miền trong của một trong ba tam giác này

44
nên theo Bổ đề trên, mỗi điểm phải các một đỉnh nào đó của ngũ giác một
khoảng không lớn hơn 1. Theo nguyên lý Dirichlet, có một đỉnh của ngũ giác
2011
có khoảng cách không lớn hơn 1 đến ít nhất [ + 1] = 403 điểm. Vậy bài
5
toán được chứng minh.
2011
Nhận xét 2.3.4. Ngay trước khi giải bài toán này, ta cần lưu ý = 402, 2
5
rất gần với 403. Điều đó gợi ý chúng ta dùng nguyên lý Dirichlet khi chỉ cả đa
giác này thành 5 phần. Vì thế mà sẽ để ý tới hoặc là 5 trung điểm của 5 cạnh
hoặc là 5 đỉnh bằng suy luận hợp lý ta thấy 5 đỉnh sẽ là lựa chọn tốt hơn.

45
Kết luận

Với mục đích nghiên cứu là: Trình bày một cách có chọn lọc về bài toán cực
trị trong hình học tổ hợp, luận văn đã hoàn thành các nhiệm vụ sau:

• Trình bày sơ lược một số nguyên lý, tính chất, phương pháp liên quan đến
việc giải các bài toán tìm giá trị lớn nhất, nhỏ nhất trong hình học tổ hợp.

• Sưu tầm, tổng hợp trình bày một số bài tập về tìm giá trị nhỏ nhất, lớn
nhất trong hình học tổ hợp là các bài toán dành cho học sinh giỏi, là đề
thi chọn học sinh giỏi trong nước và quốc tế.

• Đối với một số bài toán trong tài liệu tham khảo chưa đưa ra lời giải hoặc
chỉ có gợi ý hoặc lời giải tóm tắt, luận văn đã cố gắng đưa ra được lời giải
chi tiết để học sinh phổ thông có thể đọc, hiểu và vận dụng để giải một
số bài toán tương tự.

Hướng của luận văn là mở, khi ta tiếp tục nghiên cứu, sử dụng các kỹ thuật
giải khác để đưa thêm lời giải cho các bài toán trên.

46
Tài liệu tham khảo

[1] Vũ Hữu Bình và Sputnik Education (12/2016), Hình học tổ hợp, NXB Đại
học Quốc Gia Hà Nội.

[2] Đoàn Quỳnh - Hạ Vũ Anh – Phạm Khắc Ban – Văn Như Cường – Vũ
Đình Hòa (2008), Tài liệu chuyên toán bài tập hình học 12, NXB Giáo dục
Việt Nam.

[3] Nguyễn Bá Đăng (2005), Rèn luyện và phát triển tư duy thông qua giải
các bài toán Hình học phẳng, NXB Giáo dục.

[4] Nguyễn Hữu Điền (2005), Một số chuyên đề hình học tổ hợp, NXB Giáo
dục-Hà Nội.

[5] Vũ Hữu Bình (2016), Các bài toán về hình học tổ hợp, NXB Giáo dục.

[6] Vũ Đình Hòa (2003), Một số kiến thức cơ sở về hình học tổ hợp, NXB Giáo
dục.

[7] PGS.TS. Vũ Dương Thụy - ThS. Nguyễn Văn Nho (2001), 40 năm Olympic
toán học Quốc tế, NXB Giáo dục.

[8] Nguyễn Đình Thành Công – Nguyễn Văn Hưởng – Nguyễn Duy Hưng –
Trần Trí Kiên – Nguyễn Văn Sơn – Lê Nhất – Trần Bảo Trung (2016),
Chuyên đề bồi dưỡng học sinh giỏi qua các kỳ thi Olympic toán, NXB Đại
học Quốc gia Hà Nội.

[9] Lecturenot (September 2003), Introduction to Discrete Geometry, Depart-


ment of Applied Mathematics, Charles University Malostranské ná.25, 118
00 Praha 1, Czech Republic.

47

You might also like